exam 3 Flashcards

1
Q

Mr. O, age 50 years, comes for his annual health assessment, which is provided by his employer. During your initial history-taking interview, Mr. O mentions that he routinely engages in light exercise. At this time, you should

a. ask if he makes his own bed daily.
b. have the patient describe his exercise.
c. make a note that he walks each day.
d. record ―light exercise‖ in the history.

A

ANS: B
When Mr. O says that he engages in light exercise, have him describe his exercise. To qualify his use of the term light, ask him the type, length of time, frequency, and intensity of his activities.

How well did you know this?
1
Not at all
2
3
4
5
Perfectly
2
Q

Which of the following information belongs in the past medical history section related to heart and blood vessel assessment?

a. Adolescent inguinal hernia
b. Childhood mumps
c. History of bee stings
d. Previous unexplained fever

A

ANS: D
Previous unexplained fever should be included in the past medical history of a heart and blood vessel assessment. This incidence may be related to acute rheumatic fever, with potential heart
valve damage.

How well did you know this?
1
Not at all
2
3
4
5
Perfectly
3
Q

A patient you are seeing in the emergency department for chest pain is believed to be having a myocardial infarction. During the health history interview of his family history, he relates that
his father had died of ―heart trouble.‖ The most important follow-up question you should pose is which of the following?

a. ―Did your father have coronary bypass surgery?‖
b. ―Did your father’s father have heart trouble also?‖
c. ―What were your father’s usual dietary habits?‖
d. ―What age was your father at the time of his death?‖

A

D

A family history of sudden death, particularly in young and middle-aged relatives, significantly increases one’s chance of a similar occurrence

How well did you know this?
1
Not at all
2
3
4
5
Perfectly
4
Q

Which one of the following is a common symptom of cardiovascular disorders in the older adult?
a. Fatigue
b. Joint pain
c. Poor night vision
d. Weight gain

A

ANS: A
Common symptoms of cardiovascular disorders in older adults include confusion, dizziness, blackouts, syncope, palpitations, coughs and wheezes, hemoptysis, shortness of breath, chest
pains or tightness, impotence, fatigue, and leg edema.

How well did you know this?
1
Not at all
2
3
4
5
Perfectly
5
Q

squeezing, pressing, heavy pain

onset often occurs with exertion

severe to mild pain with a gradual build up

does not increase with inspiration or position change

may have associated symptoms of nausea & diaphoresis

A

Myocardial Ischemic Pain

How well did you know this?
1
Not at all
2
3
4
5
Perfectly
6
Q

Crushing substernal pain with radiation to neck or left arm

Pain >7

Association of exertion or stress and relief with rest

Duration of minutes

Associated symptoms: nausea, diaphoreses, weakness, SOB

A

angina

How well did you know this?
1
Not at all
2
3
4
5
Perfectly
7
Q

In the adult, the apical impulse should be most visible when the patient is in what position?
a. Supine
b. Upright
c. Lithotomy
d. Right lateral recumbent

A

ANS: B
In most adults, the apical impulse should be visible at about the midclavicular line in the fifth left intercostal space, but it is easily obscured by obesity, large breasts, or muscularity. The apical impulse may become visible only when the patient sits upright and the heart is brought closer to the anterior wall. A visible and palpable impulse when the patient is supine suggests an intensity that may be the result of a problem. In most adults, the apical impulse will not be
visible in the upright, lithotomy, or right lateral recumbent positions.

How well did you know this?
1
Not at all
2
3
4
5
Perfectly
8
Q

MI
Unstable angina
Symptoms start gradually
Last > 15 minutes
Unrelieved by rest/nitro

A

Acute coronary Syndromes

How well did you know this?
1
Not at all
2
3
4
5
Perfectly
9
Q

If the apical impulse is more vigorous than expected, it is called a
a. lift.
b. thrill.
c. bruit.
d. murmur

A

ANS: A
If the apical impulse is more vigorous than expected, it is referred to as a lift or heave. A thrill is a palpable murmur. A bruit is an auscultated arterial murmur. A murmur is an auscultated
sound caused by turbulent blood flow into, through, or out of the heart.

How well did you know this?
1
Not at all
2
3
4
5
Perfectly
10
Q

A palpable rushing vibration over the base of the heart at the second intercostal space is called a
a. heave.
b. lift.
c. thrill.
d. thrust.

A

ANS: C
A thrill is a fine, palpable, rushing vibration—a palpable murmur. Cardiac thrills generally indicate a disruption of the expected blood flow related to some defect in the closure of one of the semilunar valves (generally aortic or pulmonic stenosis), pulmonary hypertension, or atrial septal defect. A heave or lift is a more vigorous apical impulse. A lift is another term for a heave, which is a more vigorous apical impulse. A thrust is sudden, forcible forward
movement.

How well did you know this?
1
Not at all
2
3
4
5
Perfectly
11
Q

An apical PMI palpated beyond the fifth intercostal space may indicate
a. decreased cardiac output.
b. obesity.
c. left ventricular hypertrophy.
d. hyperventilation.

A

ANS: C
An apical impulse that is more forceful and widely distributed, fills systole, or is displaced laterally and downward may be indicative of left ventricular hypertrophy. Obesity, large breasts, and muscularity can obscure the visibility of the apical impulse

How well did you know this?
1
Not at all
2
3
4
5
Perfectly
12
Q

chest pain where

Can be severe pain with nausea and diaphoreses
greater at night with lying down

A

GERD/PUD

How well did you know this?
1
Not at all
2
3
4
5
Perfectly
13
Q

chest pain where
Pain is colicky and often RUQ; fever, nausea

A

Choleycystitis

How well did you know this?
1
Not at all
2
3
4
5
Perfectly
14
Q

chest pain where
Severe “boring” pain, radiates from epigastric are to the back; N&V, tachycardia, hypotension and diaphoresis;
tender abdomen

A

Pancreatitis

How well did you know this?
1
Not at all
2
3
4
5
Perfectly
15
Q

A lift along the left sternal border is most likely the result of
a. aortic stenosis.
b. atrial septal defect.
c. pulmonary hypertension.
d. right ventricular hypertrophy.

A

ANS: D
A lift along the left sternal border may be caused by right ventricular hypertrophy. A thrill indicates a disruption of the expected blood flow related to a defect in the closure of one of the semilunar valves, which is seen in aortic or pulmonic stenosis, pulmonary hypertension, or atrial septal defect.

How well did you know this?
1
Not at all
2
3
4
5
Perfectly
16
Q

History of injury, heavy lifting, contact sports

Generally in younger population with no Cardiac symptoms.

Chest pain can be severe (more localized)

Pain >’s with movement, cough and sometimes breathing

A

Chest wall pain and costochondritis

How well did you know this?
1
Not at all
2
3
4
5
Perfectly
17
Q

To estimate heart size by percussion, you should begin tapping at the
a. anterior axillary line.
b. left sternal border.
c. midclavicular line.
d. midsternal line.

A

ANS: A
Estimating the size of the heart can be done by percussion. Begin tapping at the anterior axillary line, moving medially along the intercostal spaces toward the sternal border. The change from a resonant to a dull note marks the cardiac border.

How well did you know this?
1
Not at all
2
3
4
5
Perfectly
18
Q

To hear diastolic heart sounds, you should ask patients to
a. lie on their back.
b. lie on their left side.
c. lie on their right side.
d. sit up and lean forward.

A

ANS: B
Left lateral recumbent is the best position to hear the low-pitched filling sounds in diastole with the bell of the stethoscope. Sitting up and leaning forward is the best position in which to hear relatively high-pitched murmurs with the diaphragm of the stethoscope. The right lateral recumbent position is the best position for evaluating the right rotated heart of dextrocardia

How well did you know this?
1
Not at all
2
3
4
5
Perfectly
19
Q

due to Inflammation

burning, stabbing, or cutting pain

exacerbated by coughing, deep breathing, and lying down

alleviated by leaning forward and remaining motionless

A

Pericardial Pain

How well did you know this?
1
Not at all
2
3
4
5
Perfectly
20
Q

You are listening to a patient’s heart sounds in the aortic and pulmonic areas. The sound becomes asynchronous during inspiration. The prevalent heart sound in this area is most likely
a. S1.
b. S2.
c. S3.
d. S4.

A

ANS: B
S2 marks the closure of the semilunar valves, which indicates the end of systole; it is best heard in the aortic and pulmonic areas. It is higher pitched and shorter than S1. S2 typically splits during inspiration.

How well did you know this?
1
Not at all
2
3
4
5
Perfectly
21
Q

Chest pain with an organic cause in a child is most likely the result of
a. cardiac disease.
b. asthma.
c. esophageal reflux.
d. arthritis.

A

ANS: B
Unlike chest pain in adults, chest pain in children and adolescents is seldom caused by a cardiac problem. More likely, the case is related to trauma, exercise-induced asthma, or cocaine use.

How well did you know this?
1
Not at all
2
3
4
5
Perfectly
22
Q

A condition that is likely to present with dizziness and syncope is
a. bacterial endocarditis.
b. hypertension.
c. sick sinus syndrome.
d. pericarditis.

A

ANS: C
Sick sinus syndrome (SSS) is a sinoatrial dysfunction that occurs secondary to hypertension, arteriosclerotic heart disease, or rheumatic heart disease. SSS causes dysrhythmia with
subsequent syncope, transient dizzy spells, light-headedness, seizures, palpitations, angina, or congestive heart failure (CHF). Bacterial endocarditis presents with prolonged fever, signs of
neurologic dysfunction, and sudden onset of CHF. Chest pain is an initial symptom in acute pericarditis, along with a triphasic friction rub.

How well did you know this?
1
Not at all
2
3
4
5
Perfectly
23
Q

Your patient, who abuses intravenous (IV) drugs, has a sudden onset of fever and symptoms of congestive heart failure. Inspection of the skin reveals nontender erythematic lesions to the
palms. These findings are consistent with the development of
a. rheumatic fever.
b. cor pulmonale.
c. pericarditis.
d. endocarditis.

A

ANS: D
Endocarditis is a bacterial infection of the endothelial layer of the heart. It should be suspected with at-risk patients (e.g., IV drug abusers) who present with fever and sudden onset of congestive heart symptoms. The lesions described are Janeway lesions.

How well did you know this?
1
Not at all
2
3
4
5
Perfectly
24
Q

The most helpful finding in determining left-sided heart failure is
a. dyspnea.
b. orthopnea.
c. jugular vein distention.
d. an S3 heart sound

A

ANS: C
Evidence-based research has shown that the most helpful clinical examination finding supportive of left-sided heart failure is jugular vein distention.

How well did you know this?
1
Not at all
2
3
4
5
Perfectly
25
Q

Your patient has been diagnosed with pericarditis. Which are signs and symptoms, or a precipitating factor? (Select all that apply.)
a. Sharp pain
b. Pain relieved by sitting up
c. Pain relieved by resting
d. Friction rub heard to right of sternum
e. History of kidney failure
f. Result of viral infection
g. Result of medications such as procainamide

A

ANS: A, B, E, F, G
Pericarditis may be seen with a viral infection, kidney failure, or medications such as procainamide. Symptoms include pain relieved by sitting up or leaning forward. A friction rub is heard at the left of the sternum, at the third or fourth intercostal space.

How well did you know this?
1
Not at all
2
3
4
5
Perfectly
26
Q

Induration, edema, and hyperpigmentation are common associated findings with which of the following?
a. Peripheral arterial disease
b. Venous ulcer
c. Arterial embolic disease
d. Venous thrombus

A

ANS: B
A venous ulcer also results from chronic venous insufficiency and demonstrates induration edema and hyperpigmentation. Peripheral arterial edema results in ischemia, in which the foot or leg is painful and cold; nonulceration is common as the muscles atrophy. Arterial embolic disease includes occlusion of the small arteries, resulting in blue toe syndrome and splinter hemorrhages in the nail bed. A venous thrombus presents with minimal ankle edema, low-grade fever, tachycardia, and possibly a positive Homan sign.

How well did you know this?
1
Not at all
2
3
4
5
Perfectly
27
Q

The most prominent component of the jugular venous pulse is the
a. a wave.
b. c wave.
c. v wave.
d. x slope.

A

ANS: A
The a wave is the first and most prominent component of the jugular venous pulse. The a wave represents a brief backflow of blood into the vena cava during right atrial contraction.

How well did you know this?
1
Not at all
2
3
4
5
Perfectly
28
Q

During a routine prenatal visit, Ms. T was noted as having dependent edema, varicosities of the legs, and hemorrhoids. She expressed concern about these symptoms. You explain to Ms.T that her enlarged uterus is compressing her pelvic veins and her inferior vena cava. You would further explain that these findings
a. are usual conditions during pregnancy.
b. indicate a need for hospitalization.
c. indicate the need for amniocentesis.
d. suggest that she is having twins.

A

ANS: A
Explain to the patient that these are usual conditions during pregnancy. Blood in the lower extremities tends to pool in later pregnancy because of the occlusion of the pelvic veins and
inferior vena cava from pressure created by the enlarged uterus. This occlusion results in an increase in dependent edema, varicosities of the legs and vulva, and hemorrhoids

How well did you know this?
1
Not at all
2
3
4
5
Perfectly
29
Q

Vascular changes expected in the older adult include
a. loss of vessel elasticity.
b. decreased peripheral resistance.
c. decreased pulse pressure.
d. constriction of the aorta and major bronchi.

A

ANS: A
With age, the walls of the arteries become calcified and they lose their elasticity and vasomotor tone; therefore, they lose their ability to respond appropriately to changing body needs. Increased peripheral vascular resistance occurs, causing an increase in blood pressure

How well did you know this?
1
Not at all
2
3
4
5
Perfectly
30
Q

You are examining Mr. S, a 79-year-old diabetic man complaining of claudication. Which of the following physical findings is consistent with the diagnosis of peripheral arterial disease?
a. Thick, calloused skin
b. Ruddy, thin skin
c. Warmer temperature of extremity in contrast to other body parts
d. Loss of hair over the extremities

A

ANS: D
An individual with peripheral artery disease or claudication will have thin skin with localized pallor and cyanosis, a loss of body warmth in the affected area, and loss of hair over the extremities.

How well did you know this?
1
Not at all
2
3
4
5
Perfectly
31
Q

You are performing a physical examination on a 46-year-old male patient. His examination findings include the following: positive peripheral edema, holosystolic murmur in the tricuspid region, and a pulsatile liver. His diagnosis is
a. an aortic aneurysm.
b. an arteriovenous fistula
c. tricuspid stenosis.
d. tricuspid regurgitation.

A

ANS: D
An aneurysm is a localized isolation that results in a pulsatile swelling and a thrill or bruit. An arteriovenous fistula is a pathologic communication between an artery and vein resulting in a
thrill or bruit and edema or ischemia in the involved extremity. Tricuspid regurgitation results in a holosystolic murmur in the tricuspid region, a pulsatile liver, and peripheral edema.

How well did you know this?
1
Not at all
2
3
4
5
Perfectly
32
Q

A characteristic distinguishing primary Raynaud phenomenon from secondary Raynaud phenomenon includes which of the following?
a. Vasospasm
b. Digital ischemia with pain
c. Triphasic demarcated skin
d. Cold and achy improving with warming

A

ANS: B
In primary Raynaud phenomenon, there is triphasic demarcation of the skin—white, cyanotic, and reperfused—and vasospasm that lasts a minutes to less than an hour, areas of cold, and an achy feeling that improves with rewarming. In secondary Raynaud phenomenon, there is intense pain from digital ischemia

How well did you know this?
1
Not at all
2
3
4
5
Perfectly
33
Q

In children, coarctation of the aorta should be suspected if you detect
a. a delay between the radial and femoral pulses.
b. a simultaneous radial and femoral pulse.
c. an absent femoral pulse on the left.
d. bilateral absence of femoral pulses.

A

ANS: A
Coarctation of the aorta is a congenital stenosis or narrowing seen most commonly in the descending aortic arch, near the origin of the left subclavian artery and ligamentum
arteriosum. Ordinarily, the radial and femoral pulses are palpated simultaneously. When there is a delay and/or a palpable diminution of amplitude of the femoral pulse, coarctation must be suspected. Differences in blood pressure taken in the arms and legs should confirm the suspicion. Coarctation of the aorta should not be suspected if the radial and femoral pulses are palpated simultaneously, if the femoral pulse on the left is absent, or if there is bilateral absence of femoral pulses.

How well did you know this?
1
Not at all
2
3
4
5
Perfectly
34
Q

Which of the following statements is true regarding the development of venous ulcers in older adults?
a. The major symptom is severe leg pain, especially when walking.
b. The affected leg is commonly pale and hairless, and pulses are difficult to palpate.
c. Diabetes, peripheral neuropathy, and nutritional deficiencies are causative factors.

A

ANS: C
Venous ulcers are generally found on the medial or lateral aspects of the lower limbs, most often in older adults. Induration, edema, and hyperpigmentation are common. Heart failure, hypoalbuminemia, peripheral neuropathy, diabetes mellitus, nutritional deficiencies, and
arterial disease cause the venous ulcers to develop. The major symptom of venous ulcers is not severe leg pain. In patients with venous ulcers, the affected leg is not commonly pale and
hairless, and pulses are not difficult to palpate. Venous ulcers are not generally located on the tips of toes.

How well did you know this?
1
Not at all
2
3
4
5
Perfectly
35
Q

When examining arterial pulses, the thumb may be used
a. especially if vessels have a tendency to move.
b. never for palpating pulses.
c. checking the jugular venous pressure.
d. during the Allen test

A

ANS: A
The thumb may be used, especially if the vessels have a tendency to move when probed by the fingers. The thumb is particularly useful in fixing the brachial and even the femoral pulses. You cannot palpate for jugular venous pressure waves. The Allen test is used to ensure ulnar patency prior to radial artery puncture.

How well did you know this?
1
Not at all
2
3
4
5
Perfectly
36
Q

To assess a patient’s jugular veins, the patient should first be placed in which position?
a. Supine
b. Semi-Fowler
c. Upright
d. Left lateral recumbent

A

ANS: A
To assess jugular veins, place the patient in the supine position. This causes engorgement of the jugular veins. Then gradually raise the head of the bed until the pulsations of the jugular vein become visible between the angle of the jaw and the clavicle. Jugular veins cannot be palpated.

How well did you know this?
1
Not at all
2
3
4
5
Perfectly
37
Q

Observation of hand veins can facilitate the assessment of
a. mitral valve competency.
b. a heart murmur.
c. right heart pressure.
d. left heart pressure.

A

ANS: C
Hand veins can be used as an auxiliary manometer of right heart pressure. Assess the hand veins while the hand is at the patient’s side. Then raise the hand until the veins collapse, and use a ruler to measure the vertical distance between the midaxillary line (level of the heart) and the level of the collapsed hand veins.

How well did you know this?
1
Not at all
2
3
4
5
Perfectly
38
Q

You are assessing Mr. Z’s fluid volume status as a result of heart failure. If your finger depresses a patient’s edematous ankle to a depth of 6 mm, you should record this pitting as
a. 1+.
b. 2+.
c. 3+.
d. 4+.

A

ANS: C
Pitting edema to 6 mm represents a 3+ rating. This edema is noticeably deep and may last more longer a minute; the dependent extremity looks fuller and swollen. Edema is graded on a
scale of mild (1+) through worse (4+).

How well did you know this?
1
Not at all
2
3
4
5
Perfectly
39
Q

A bounding pulse in an infant may be associated with
a. patent ductus arteriosus.
b. coarctation of the aorta.
c. decreased cardiac output.
d. peripheral vaso-occlusion.

A

ANS: A
A bounding pulse is associated with a large left-to-right shunt produced by a patent ductus arteriosus. A weaker or thinner pulse represents diminished cardiac output or peripheral vasoconstriction. A difference in pulse amplitude between the upper extremities or between the femoral and radial pulses, and absence of the femoral pulse, suggests a coarctation of the aorta.

How well did you know this?
1
Not at all
2
3
4
5
Perfectly
40
Q

In infants or small children, a capillary refill time of 4 seconds
a. is normal.
b. indicates hypervolemia.
c. indicates dehydration or hypovolemic shock.
d. indicates renal artery stenosis.

A

c
Capillary refill time represents the time it takes the capillary bed to refill after being occluded by pressure to the nail bed for several seconds. Observe the time it takes for the nail to regain
its full color, which should be less than 2 seconds for an intact system. The capillary refill time will be longer than 2 seconds during arterial occlusion, hypovolemic shock, hypothermia, and dehydration.

How well did you know this?
1
Not at all
2
3
4
5
Perfectly
41
Q

A venous hum heard over the internal jugular vein of a child
a. usually signifies untreatable illness.
b. usually has no pathologic significance.
c. usually requires surgical intervention.
d. must be monitored until the child is grown.

A

ANS: B
A venous hum is caused by the turbulence of blood flow in the internal jugular veins. It is common in children and usually has no pathologic significance. To detect a venous hum, auscultate over the right supraclavicular space at the medial end of the clavicle and along the anterior border of the sternocleidomastoid muscle. It is louder during diastole.

How well did you know this?
1
Not at all
2
3
4
5
Perfectly
42
Q

You are palpating bilateral pedal pulses and cannot feel one of the pulses. The feet are equally warm. You find that both great toes are pink, with a capillary refill within 2 seconds. Which of the following statements is correct?
a. Immediate emergency surgery is indicated.
b. Pedal pulses are not always palpable.
c. Unilateral pulses are never normal.
d. Venogram studies will be needed.

A

ANS: B
Dorsalis pedis and posterior tibia pulses may be difficult to palpate or may not be palpable in some well persons. The feet are warm and capillary refill is less than 2 seconds; there is
adequate circulation to the feet. Immediate emergency surgery is not indicated. Unilateral pulses may be normal. Venogram studies will not be needed.

How well did you know this?
1
Not at all
2
3
4
5
Perfectly
43
Q

When palpating the carotid artery, which of the following is most important? (Select all that apply.)
a. Rotate the patient’s head to the side being examined to relax the
sternocleidomastoid.
b. Excessive carotid sinus massage can compromise blood flow to the brain.
c. Excessive carotid sinus massage can cause slowing of the pulse.
d. Palpate both sides simultaneously.

A

ANS: A, B, C
When palpating the carotid arteries, never palpate both sides simultaneously. Excessive carotid sinus massage can cause slowing of the pulse and a drop in blood pressure and can compromise blood flow to the brain, leading to syncope. If you have difficulty feeling the pulse, rotate the patient’s head to the side being examined to relax the sternocleidomastoid muscle

How well did you know this?
1
Not at all
2
3
4
5
Perfectly
44
Q

Which are risk factors for varicose veins? (Select all that apply.)
a. Gender
b. Alcohol use
c. Lower extremity trauma
d. Increased body mass
e. Hypertension
f. Diabetes

A

ANS: A, C, D
Gender (women are four times more likely than men to have varicose veins—genetic predisposition), tobacco use, increased body mass, age, and history of lower extremity trauma are all risk factors for varicose veins.

How well did you know this?
1
Not at all
2
3
4
5
Perfectly
45
Q

S1 increases in intensity with

A

high velocity states
mitral valve stenosis

How well did you know this?
1
Not at all
2
3
4
5
Perfectly
46
Q

S1 decreases in intensity
with

A

emphysema
obesity

How well did you know this?
1
Not at all
2
3
4
5
Perfectly
47
Q

S2 increases intensity
with

A

HTN, exercise

How well did you know this?
1
Not at all
2
3
4
5
Perfectly
48
Q

S2 decreases in intensity
with

A

obese, hypotension

How well did you know this?
1
Not at all
2
3
4
5
Perfectly
49
Q

louder at apex
coincides with carotid pulse

A

s1

How well did you know this?
1
Not at all
2
3
4
5
Perfectly
50
Q

type of abd pain that
diffuse & midline

results in restlessness

A

Visceral pain:

How well did you know this?
1
Not at all
2
3
4
5
Perfectly
51
Q

type of abd pain that localized & sharp

movement exacerbates pain

A

Parietal pain:

How well did you know this?
1
Not at all
2
3
4
5
Perfectly
52
Q

type of abd pain that
esophagus/stomach

A

Epigastric pain:

How well did you know this?
1
Not at all
2
3
4
5
Perfectly
53
Q

RUQ: abd pain can be what 3 things

A

gallbladder, thorax, liver

How well did you know this?
1
Not at all
2
3
4
5
Perfectly
54
Q

LUQ:
pain can be

A

spleen

How well did you know this?
1
Not at all
2
3
4
5
Perfectly
55
Q

RLQ: pain can be

A

PID, appendicitis, ectopic pregnancy

How well did you know this?
1
Not at all
2
3
4
5
Perfectly
56
Q

LLQ abd pain can be

A

LLQ: sigmoid colon

How well did you know this?
1
Not at all
2
3
4
5
Perfectly
57
Q

abd Pain with deep inspiration-

A

pleuritic pain or biliary colic

How well did you know this?
1
Not at all
2
3
4
5
Perfectly
58
Q

Infectious causes most common
self limited
abrupt onset that resolves within 3 weeks
associated with pain, N&V, fever

A

Acute
diarrhea

How well did you know this?
1
Not at all
2
3
4
5
Perfectly
59
Q

lasts > 4 weeks
parasites, medication
IBS or inflammatory bowel disease
lactose intolerance

A

chronic diarrhea

How well did you know this?
1
Not at all
2
3
4
5
Perfectly
60
Q

5 red flags for abd pain

A

**Progressive and persistent pain
Progressive distention
Hematemesis
Black tarry stool
Decreased urine output
**

How well did you know this?
1
Not at all
2
3
4
5
Perfectly
61
Q

In the absence of trauma or blood disorders, the appearance of ________is a valuable bedside clue to retroperitoneal or intraperitoneal hemorrhage.

A

Cullen’s sign

How well did you know this?
1
Not at all
2
3
4
5
Perfectly
62
Q

refers to bruising of the flanks.

This sign takes 24–48 hours. It can predict a severe attack of acute pancreatitis, with mortality rising from 8-10% to 40%.

It may be accompanied by Cullen’s sign, which may then be indicative of pancreatic necrosis with retroperitoneal or intraabdominal bleeding.

A

Grey Turner’s sign

How well did you know this?
1
Not at all
2
3
4
5
Perfectly
63
Q

can include entire GI tract, ulcers, fissures, discontinuous and stepwise lesions.

Cobblestone appearance

A

Crohn’s disease

How well did you know this?
1
Not at all
2
3
4
5
Perfectly
64
Q

continuous inflammation from rectum proximal
tenesmus
Surgery to remove damaged colon

A

Ulcerative Colitis

How well did you know this?
1
Not at all
2
3
4
5
Perfectly
65
Q

Noninflammatory enlargement of male breast tissue.

It is physiologic at puberty, unilateral, usually mild and transient.

Occurs commonly in aging men due to changing hormone levels.

It is usually bilateral and may be tender.

Can occur with exogenous hormone: estrogen for prostate cancer, anabolic steroids, marijuana, thryrotoxicosis,

A

Gynecomastia

How well did you know this?
1
Not at all
2
3
4
5
Perfectly
66
Q

Solid tumor
Men age 20-40

Risk factors
Cryptoorchidism
Caucasian
FH of testicular cancer

S&S
Firm painless mass
c/o “heavy feeling in scrotum”
No transillumination

A

testicular cancer

How well did you know this?
1
Not at all
2
3
4
5
Perfectly
66
Q

Inflammation of testicles

Causes
Sex, infection, instrumentation
Gradual onset
One sided
Urinary S&S
Urethral discharge

PE
Tender testes
Swelling
Testes are normal

A

Epididymitis

How well did you know this?
1
Not at all
2
3
4
5
Perfectly
67
Q

Scrotal swelling
Shiny, without rugae
Transillumination

A

Hydrocele

How well did you know this?
1
Not at all
2
3
4
5
Perfectly
68
Q

Emergency
Twisting of spermatic cord
Sudden onset of pain
Radiates groin
N&V

NO urinary S&S, not related to sex, no discharge

A

Testicular tortion

How well did you know this?
1
Not at all
2
3
4
5
Perfectly
69
Q

Aging

Irritative S&S
Frequency, nocturia, urgency

Obstructive S&S
Weak urinary stream, < force of stream, interrupted stream

PE
Firm, smooth, obliterated sulcus

Health maintenance
AUA screen p. 260 Dains

A

BPH

How well did you know this?
1
Not at all
2
3
4
5
Perfectly
70
Q

Inflammation
Bacterial/nonbacterial
Acute
u/a
> WBC and bacteria
Tenderness
Systemic S&S
Fever, chills, malaise
Low back pain
Urinary S&S
Pain
Peri area, suprapubic,
Sexual dysfunction
pain with sex

Chronic
Gradual onset of urinary S&S
May have history of recurrent & difficult to treat UTI

A

Prostatitis

How well did you know this?
1
Not at all
2
3
4
5
Perfectly
71
Q

Enlarged prostate
Biopsy
PSA
Ultrasound

A

Prostate Cancer

How well did you know this?
1
Not at all
2
3
4
5
Perfectly
72
Q

Infects urethra in men
Painful urination
Dyspareunia
Painful testis

Discharge
Silent epidemic in women
75% have no symptoms

A

Chlamydia

How well did you know this?
1
Not at all
2
3
4
5
Perfectly
73
Q

The family history of a patient with diarrhea and abdominal pain should include inquiry about cystic fibrosis because it is

a. a common genetic disorder.
b. one cause of malabsorption syndrome.
c. a curable condition with medical intervention.
d. the most frequent cause of diarrhea in general practice.

A

ANS: B
Cystic fibrosis is an uncommon, chronic genetic disorder affecting multiple systems. In the gastrointestinal tract, it causes malabsorption syndrome because of pancreatic lipase
deficiency. Steatorrhea and abdominal pain from increased gas production are frequent complaints.

How well did you know this?
1
Not at all
2
3
4
5
Perfectly
74
Q

Mrs. James is 7 months’ pregnant and states that she has developed a problem with constipation. She eats a well-balanced diet and is usually regular. You should explain that constipation is common during pregnancy because of changes in the colorectal areas, such as
a. decreased movement through the colon and increased water absorption from the
stool.
b. increased movement through the colon and increased salt taken from foods.
c. looser anal sphincter and fewer nutrients taken from foods.
d. tighter anal sphincter and less iron eliminated in the stool.

A

ANS: A
Constipation and flatus are more common during pregnancy because the colon is displaced, peristalsis is decreased, and water absorption is increased. Movement through the colon is decreased during pregnancy. The colon does not absorb nutrients. A tighter sphincter tone is not related to pregnancy

How well did you know this?
1
Not at all
2
3
4
5
Perfectly
75
Q

The family history of a patient with diarrhea and abdominal pain should include inquiry about cystic fibrosis because it is
a. a common genetic disorder.
b. one cause of malabsorption syndrome.
c. a curable condition with medical intervention.
d. the most frequent cause of diarrhea in general practice.

A

ANS: B
Cystic fibrosis is an uncommon, chronic genetic disorder affecting multiple systems. In the gastrointestinal tract, it causes malabsorption syndrome because of pancreatic lipase
deficiency. Steatorrhea and abdominal pain from increased gas production are frequent complaints.

How well did you know this?
1
Not at all
2
3
4
5
Perfectly
76
Q

When assessing abdominal pain in a college-age woman, one must include
a. history of interstate travel
b. food likes and dislikes.
c. age at completion of toilet training.
d. the first day of the last menstrual period.

A

ANS: D
Exploring abdominal pain complaints in a young woman can reveal multiple causes related to the menstrual cycle, including menstrual pain, ovulation discomfort, and abnormal menses. Asking the patient to tell you the first day of her last menstrual period can help discriminate among these factors. History of international travel and traveler’s diarrhea can be related to abdominal pain, but interstate travel usually does not. Food preferences and age at completion
of toilet training are not relevant

How well did you know this?
1
Not at all
2
3
4
5
Perfectly
77
Q

Infants born weighing less than 1500 g are at higher risk for
a. hepatitis A.
b. necrotizing enterocolitis.
c. urinary urgency.
d. pancreatitis.

A

ANS: B
Necrotizing enterocolitis is a gastrointestinal disease that mostly affects premature infants. It involves infection and inflammation that cause destruction of the bowel, and it becomes more apparent after feedings.

How well did you know this?
1
Not at all
2
3
4
5
Perfectly
78
Q

You are completing a general physical examination on Mr. Rock, a 39-year-old man with complaints of constipation. When examining a patient with tense abdominal musculature, a helpful technique is to have the patient

a. hold his or her breath.
b. sit upright.
c. flex his or her knees.
d. raise his or her head off the pillow.

A

ANS: C
To help relax the abdominal musculature, it is helpful to place a small pillow under the patient’s head and under slightly flexed knees. The other techniques are not helpful because they increase muscle flexion.

How well did you know this?
1
Not at all
2
3
4
5
Perfectly
79
Q

Mrs. Little is a 44-year-old patient who presents to the office with abdominal pain and fever.During your examination, you ask the patient to raise her head and shoulders while she lies in a supine position. A midline abdominal ridge rises. You document this observation as a(n)

a. small inguinal hernia.
b. large epigastric hernia.
c. abdominal lipoma.
d. diastasis recti.

A

ANS: D
A diastasis recti occurs when the abdominal contents bulge between two abdominal muscles to form a midline ridge as the head is lifted. It has little clinical significance and usually occurs in women who have had repeated pregnancies and in obese patients.

How well did you know this?
1
Not at all
2
3
4
5
Perfectly
80
Q

Mr. Robins is a 45-year-old man who presents to the emergency department with a complaint of constipation. During auscultation, you note borborygmi sounds. This is associated with
a. gastroenteritis.
b. peritonitis.
c. satiety.
d. paralytic ileus.

A

ANS: A
Borborygmi are prolonged loud gurgles that occur with gastroenteritis, early intestinal
obstruction, or hunger. Peritonitis and paralytic ileus result in hypoactive bowel sounds. Food
satiety does not stimulate growling sounds as does hunger.

How well did you know this?
1
Not at all
2
3
4
5
Perfectly
81
Q

To document absent bowel sounds correctly, one must listen continuously for
a. 30 seconds.
b. 1 minute.
c. 3 minutes.
d. 5 minutes.

A

ANS: D
Absent bowel sounds are confirmed after listening to each quadrant for 5 minutes.

How well did you know this?
1
Not at all
2
3
4
5
Perfectly
82
Q

Percussion of the abdomen begins with establishing
a. liver dullness.
b. spleen dullness.
c. gastric bubble tympany.
d. overall dullness and tympany in all quadrants.

A

ANS: D
Percussion begins with a general establishment over all quadrants for areas of dullness and tympany and then proceeds to specific target organs.

How well did you know this?
1
Not at all
2
3
4
5
Perfectly
83
Q

When percussing a spleen, Traube space is a
a. semilunar region.
b. splenic percussion sign.
c. left-sided pleural effusion.
d. solid mass.

A

ANS: A
Percussion of the spleen is more difficult because percussion tones elicited may be caused by other conditions. Traube space is a semilunar region defined by the sixth ribs superiorly, the
midaxillary line laterally, and the left costal margin inferiorly.

How well did you know this?
1
Not at all
2
3
4
5
Perfectly
84
Q

Your patient is complaining of acute, intense, sharp epigastric pain that radiates to the back and left scapula, with nausea and vomiting. Based on this history, your prioritized physical examination should be to
a. percuss for ascites.
b. assess for rebound tenderness.
c. inspect for ecchymosis of the flank.
d. auscultate for abdominal bruits.

A

ANS: C
Abdominal pain that radiates to the back could be caused by pancreatitis or a gastric ulcer, gallbladder pain usually radiates to the right or left scapula but not to the back, pancreatitis pain can radiate to the left shoulder or scapula, and nausea and vomiting usually occur with gallbladder, pancreas, or appendix conditions. Pancreatitis is a differential diagnosis for all these symptoms, so begin the examination by inspecting the flanks for the Grey Turner sign, an indication of pancreatitis

How well did you know this?
1
Not at all
2
3
4
5
Perfectly
85
Q

To assess for liver enlargement in the obese person, you should
a. use the hook method.
b. have the patient lean over at the waist.
c. auscultate using the scratch technique.
d. attempt palpation during deep exhalation.

A

ANS: C
If the abdomen is obese or distended, or if the abdominal muscles are tight, you should plan on auscultating the liver using the scratch method to estimate the lower border of the liver.

How well did you know this?
1
Not at all
2
3
4
5
Perfectly
86
Q

Your patient presents with symptoms that lead you to suspect acute appendicitis. Which assessment finding is least likely to be associated with this condition?
a. Positive psoas sign
b. Positive McBurney sign
c. Consistent right lower quadrant (RLQ) pain
d. Rebound tenderness

A

ANS: C
A positive psoas sign, McBurney point pain, rebound tenderness, and periumbilical pain that migrates to the RLQ are signs of appendicitis. The absence of pain migration makes appendicitis less likely.

How well did you know this?
1
Not at all
2
3
4
5
Perfectly
87
Q

. When using the bimanual technique for palpating the abdomen, you should
a. push down with the bottom hand and the other hand on top.
b. push down with the top hand and concentrate on sensation with the bottom hand.
c. place the hands side by side and push equally.
d. place one hand anteriorly and the other hand posteriorly, squeezing the hands
together.

A

ANS: B
The bimanual technique uses one hand on top of the other. Exert pressure with the top hand while concentrating on sensation with the other hand.

How well did you know this?
1
Not at all
2
3
4
5
Perfectly
88
Q

Flatulence, diarrhea, dysuria, and tenderness with abdominal palpation are findings usually associated with
a. diverticulitis.
b. pancreatitis.
c. ruptured ovarian cyst.
d. splenic rupture.

A

ANS: A
Only diverticulitis has all these presenting symptoms.

How well did you know this?
1
Not at all
2
3
4
5
Perfectly
89
Q

A 51-year-old woman calls with complaints of weight loss and constipation. She reports enlarged hemorrhoids and rectal bleeding. You advise her to

a. use a topical, over-the-counter hemorrhoid treatment for 1 week.
b. exercise and eat more fiber.
c. come to the laboratory for a stool guaiac test.
d. eat six small meals a day.

A

ANS: C
Blood in the stool is an abnormal finding that should never be ignored, even if it can be explained by conditions other than colon cancer. She should have her stool checked for blood
now as well as annually because she is older than 50 years.

How well did you know this?
1
Not at all
2
3
4
5
Perfectly
90
Q

Costovertebral angle tenderness should be assessed whenever you suspect that the patient may have
a. cholecystitis.
b. pancreatitis.
c. pyelonephritis.
d. ulcerative colitis.

A

ANS: C
Pyelonephritis is characterized by flank pain and costovertebral angle tenderness.

How well did you know this?
1
Not at all
2
3
4
5
Perfectly
91
Q

In older adults, overflow fecal incontinence is commonly caused by
a. malabsorption.
b. parasitic diarrhea.
c. fecal impaction
d. fistula formation.

A

ANS: C
Constipation with overflow occurs when the rectum contains hard stool and soft feces above a
leak around the mass of stool.

How well did you know this?
1
Not at all
2
3
4
5
Perfectly
92
Q

Your patient is a 48-year-old woman with complaints of severe cramping pain in the abdomen and right flank. Her past medical history includes a history of bladder calculi. You diagnose
her with renal calculi at this time. Which of the following symptoms would you expect with her diagnosis? (Select all that apply.)

a. Abdominal pain on palpation
b. Blumberg sign
c. Cullen sign
d. CVA tenderness
e. Fever
f. Grey Turner sign
g. Hematuria
h. Nausea

A

ANS: A, D, E, G
Abdominal pain on palpation, CVA tenderness, fever, hematuria, and nausea are all signs and symptoms of renal calculi. The Cullen sign is ecchymosis around the umbilicus, the Blumberg sign is rebound tenderness for appendicitis, the Grey Turner sign is ecchymosis in the flanks, and the McBurney sign is rebound tenderness at McBurney’s point.

How well did you know this?
1
Not at all
2
3
4
5
Perfectly
93
Q

Your patient returns to the office with multiple complaints regarding her abdomen. Which of the following are objective findings? (Select all that apply.)
a. Nausea
b. Dullness on percussion
c. Rebound tenderness
d. Vomiting
e. Diarrhea
f. Burning pain in epigastrium

A

ANS: B, C, E, F
Nausea, vomiting and diarrhea, and burning pain in epigastrium are subjective signs. Dullness on percussion and rebound tenderness are objective findings.

How well did you know this?
1
Not at all
2
3
4
5
Perfectly
94
Q

In an uncircumcised male, retraction of the foreskin may reveal a cheesy white substance. This is usually
a. evidence of a fungal infection.
b. a collection of sebaceous material.
c. indicative of penile carcinoma.
d. suggestive of diabetes.

A

ANS: B
In the uncircumcised male, smegma is formed by the secretion of sebaceous material by the glans and the desquamation of epithelial cells from the prepuce. It appears as a cheesy white substance on the glans and in the fornix of the foreskin. Smegma lubricates the cavity between the foreskin of the penis and the glans, allowing smooth movement between them during intercourse. It is not usually evidence of a fungal infection, penile carcinoma, or diabetes

How well did you know this?
1
Not at all
2
3
4
5
Perfectly
95
Q

Inspection of the scrotum should reveal
a. lightly pigmented skin.
b. two testes per sac.
c. smooth scrotal sacs.
d. the left scrotal sac lower than the right

A

ANS: D
The left cord is longer than the right; consequently, the left testis hangs somewhat lower. The skin of the scrotum is more darkly pigmented. The scrotum has one testis per sac. The scrotum has small epidermoid cysts that give it a lumpy appearance

How well did you know this?
1
Not at all
2
3
4
5
Perfectly
96
Q

Expected genitalia changes that occur as men age include that
a. the ejaculatory volume decreases with age.
b. erections develop more quickly.
c. the viability of sperm increases.
d. the scrotum becomes more pendulous

A

ANS: D
Ejaculatory volume increases with age, erections develop more slowly, sperm viability decreases, and the scrotum becomes more pendulous with age.

How well did you know this?
1
Not at all
2
3
4
5
Perfectly
97
Q

Inspection of the male urethral orifice requires the examiner to
a. ask the patient to bear down.
b. insert a small urethral speculum.
c. press the glans between the thumb and forefinger.
d. transilluminate the penile shaft

A

ANS: C
Inspection of the urethral orifice is accomplished by pressing the glans between the examiner’s thumb and forefinger. This maneuver opens the slitlike orifice for further inspection.

How well did you know this?
1
Not at all
2
3
4
5
Perfectly
98
Q

You are inspecting the genitalia of an uncircumcised adult male. The foreskin is tight and cannot be easily retracted. You should
a. chart the finding as paraphimosis.
b. inquire about previous penile infections.
c. retract the foreskin firmly.
d. transilluminate the glans.

A

ANS: B
This condition is phimosis and is usually congenital, or it may be related to recurrent infections or poorly controlled diabetes. You should not chart this finding as paraphimosis. Retracting the foreskin forcibly would lead to further adhesion formation and worsening
phimosis. Transillumination is indicated for masses of the scrotum.

How well did you know this?
1
Not at all
2
3
4
5
Perfectly
99
Q

Which type of hernia lies within the inguinal canal?
a. Umbilical
b. Direct
c. Indirect
d. Femoral

A

ANS: C
Hernias found within the inguinal canal are called indirect hernias

How well did you know this?
1
Not at all
2
3
4
5
Perfectly
100
Q

Which condition is of minor consequence in an adult male?
a. Adhesions of the foreskin
b. Continuous penile erection
c. Lumps in the scrotal skin
d. Venous dilation in the spermatic cord

A

ANS: C
Lumps in the scrotal skin are related to numerous sebaceous cysts and are within normal limits.

How well did you know this?
1
Not at all
2
3
4
5
Perfectly
101
Q

Mr. L has an unusually thick scrotum, with edema and pitting. He has a history of cardiac problems. The appearance of his scrotum is most likely a(n)
a. congenital defect that has worsened.
b. indication of general fluid retention.
c. normal consequence of aging.
d. complication of the development of mumps.

A

b
General fluid retention can cause scrotal thickening and pitting edema, and is usually seen as a result of cardiac, renal, or hepatic disease. This swelling does not imply a condition of the genitalia, but rather a condition of these related systems.

How well did you know this?
1
Not at all
2
3
4
5
Perfectly
102
Q
A
How well did you know this?
1
Not at all
2
3
4
5
Perfectly
103
Q

A characteristic related to syphilis or diabetic neuropathy is testicular
a. dropping, with asymmetry.
b. enlargement.
c. insensitivity to painful stimulation.
d. recession into the abdomen.

A

ANS: C
Diabetic neuropathy or syphilis can cause a marked reduction of tactile perceptions. Asymmetry is a normal finding; enlargement and recession are not related to diabetes or syphilis

How well did you know this?
1
Not at all
2
3
4
5
Perfectly
104
Q

On palpation, a normal vas deferens should feel
a. beaded.
b. smooth.
c. ridged.
d. spongy.

A

ANS: B
The vas deferens should feel smooth and discrete as it is palpated from the testicle to the inguinal ring. A beaded or lumpy vas deferens might indicate diabetes or the presence of old inflammatory changes.

How well did you know this?
1
Not at all
2
3
4
5
Perfectly
105
Q

A premature infant’s scrotum will appear
a. bifid.
b. loose.
c. ridged.
d. smooth.

A

ANS: D
The premature male scrotum will appear underdeveloped, smooth, without rugae, and without testes; the full-term infant should have a loose, pendulous scrotum, with rugae and a midline
raphe

How well did you know this?
1
Not at all
2
3
4
5
Perfectly
106
Q

An enlarged painless testicle in an adolescent or adult male may indicate
a. epididymitis.
b. testicular torsion.
c. a tumor.
d. an undescended testicle.

A

ANS: C
A hard, enlarged, painless testicle can indicate a tumor in the adolescent or adult male. Epididymitis and torsion are painful; an undescended testicle is common in infants and is
usually resolved by 12 months.

How well did you know this?
1
Not at all
2
3
4
5
Perfectly
107
Q

You palpate a soft, slightly tender mass in the right scrotum of an adult male. You attempt to reduce the size of the mass, and there is no change in the mass size. Your next assessment maneuver is to

a. use two fingers to attempt to reduce the mass.
b. palpate the left scrotum simultaneously.
c. lift the right testicle and then compare pain level.
d. transilluminate the mass.

A

ANS: D
A soft mass is a hernia or hydrocele. If the mass can be reduced, it is probably a hernia; a nonreducible mass should be transilluminated to determine whether it contains fluid and is possibly caused by a hydrocele. Lifting the scrotum should be done when epididymitis is
suspected.

How well did you know this?
1
Not at all
2
3
4
5
Perfectly
108
Q

The most common cancer in young men ages 15 to 30 years is
a. testicular.
b. penile.
c. prostate.
d. anal.

A

ANS: A
Because testicular tumors are the most common cancer occurring in young adults, self-examination is encouraged.

How well did you know this?
1
Not at all
2
3
4
5
Perfectly
109
Q

The most emergent cause of testicular pain in a young male is
a. testicular torsion.
b. epididymitis.
c. tumor.
d. hydrocele

A

ANS: A
Testicular torsion is a surgical emergency. If surgery is performed within 12 hours after the onset of symptoms, the testis can be saved in about 90% of cases. Delayed treatment results in a much lower salvage rate.

How well did you know this?
1
Not at all
2
3
4
5
Perfectly
110
Q

An adolescent male is being seen for acute onset of left testicular pain. The pain started 3 hours ago. He complains of nausea and denies dysuria or fever. Your priority action should be to

a. obtain urine and DNA probe urethral samples.
b. lift the left scrotum to confirm epididymitis.
c. establish absent cremasteric reflex.
d. transilluminate the left and right scrotum.

A

ANS: C
The patient is displaying symptoms of testicular torsion. An absent cremasteric reflex is a supporting finding to differentiate torsion from epididymitis.

How well did you know this?
1
Not at all
2
3
4
5
Perfectly
111
Q

. The most common type of hernia occurring in young males is
a. hiatal.
b. incarcerated femoral.
c. indirect inguinal.
d. umbilical

A

ANS: C
The most common type of hernia in children and young males is an indirect inguinal hernia.

How well did you know this?
1
Not at all
2
3
4
5
Perfectly
112
Q

Difficulty replacing the retracted foreskin of the penis to its normal position is called
a. paraphimosis.
b. Peyronie disease.
c. phimosis.
d. priapism

A

ANS: A
Paraphimosis refers to the inability to replace the foreskin to its original position after it has been retracted behind the glans.

How well did you know this?
1
Not at all
2
3
4
5
Perfectly
113
Q

Which genital virus infection is known to have a latent phase followed by the production of viral DNA capsids and particles?
a. Condyloma acuminatum
b. Molluscum contagiosum
c. Herpetic lesions
d. Lymphogranuloma venereum

A

ANS: A
Condyloma acuminatum (genital warts) are soft, reddish lesions commonly present on the prepuce, glans penis, and shaft. These lesions can undergo latency, followed by viral DNA capsids and particles, which are produced in the host cells.

How well did you know this?
1
Not at all
2
3
4
5
Perfectly
114
Q

Pearly gray, smooth, dome-shaped, often umbilicated lesions of the glans penis are probably
a. herpetic lesions.
b. condylomata.
c. molluscum contagiosum.
d. chancres

A

ANS: C
Smooth, dome-shaped lesions with an umbilicated center of a pearly gray color are indicative of molluscum contagiosum.

How well did you know this?
1
Not at all
2
3
4
5
Perfectly
115
Q

A 12-year-old boy relates that his left scrotum has a soft swollen mass. The scrotum is not painful on palpation. The left inguinal canal is without masses. The mass transilluminates with a penlight. This collection of symptoms is consistent with
a. orchitis.
b. a hydrocele.
c. a rectocele.
d. a scrotal hernia

A

ANS: B
A hydrocele is a soft scrotal mass that occurs as a result of fluid accumulation and therefore transilluminates. Orchitis results in a swollen, tender testis. A rectocele does not result in scrotal swelling. A scrotal hernia would also be palpable along the inguinal canal.

How well did you know this?
1
Not at all
2
3
4
5
Perfectly
116
Q

Which condition is a complication of mumps in the adolescent or adult?
a. Cystitis
b. Epididymitis
c. Orchitis
d. Paraphimosis

A

ANS: C
Orchitis is uncommon unless seen as a complication of mumps in the adolescent or adult.

How well did you know this?
1
Not at all
2
3
4
5
Perfectly
117
Q

Parents of a 6-year-old boy should be asked if he has
a. erections.
b. nocturnal emissions.
c. rapid detumescence.
d. scrotal swelling.

A

ANS: D
Scrotal swelling, especially with crying or with bowel movements, signals the presence of a hernia. The questions about erections and rapid detumescence are for the older male. The question about nocturnal emissions is asked of adolescents.

How well did you know this?
1
Not at all
2
3
4
5
Perfectly
118
Q

. The male with Peyronie disease will usually complain of
a. painful, inflamed testicles.
b. deviation of the penis during erection.
c. lack of sexual interest.
d. painful lesions of the penis.

A

ANS: B
Peyronie disease is characterized by a fibrous band in the corpus cavernous. It results in unilateral deviation of the penis during erection

How well did you know this?
1
Not at all
2
3
4
5
Perfectly
119
Q

. A cremasteric reflex should result in
a. testicular and scrotal rise on the stroked side.
b. penile deviation to the left side.
c. bilateral elevation of the scrotum.
d. immediate erection of the penis.

A

ANS: A
On stroking the inner thigh with a blunt instrument or finger, the testicle and scrotum should rise on the stroked side.

How well did you know this?
1
Not at all
2
3
4
5
Perfectly
120
Q
A
How well did you know this?
1
Not at all
2
3
4
5
Perfectly
121
Q

In males, which surface of the prostate gland is accessible by digital examination?
a. Median lobe
b. Posterior
c. Superior
d. Anterior

A

ANS: B
The posterior surface of the prostate gland lies close to the anterior wall of the rectum and is palpable through digital rectal examination

How well did you know this?
1
Not at all
2
3
4
5
Perfectly
122
Q

The prostatic sulcus
a. divides the right and left lateral lobes.
b. is the site of the seminal vesicle emergence.
c. refers to the anterior aspect of the prostate.
d. secretes clear viscous mucus

A

ANS: A
The prostatic sulcus divides the two lateral lobes and is palpated as a shallow groove.

How well did you know this?
1
Not at all
2
3
4
5
Perfectly
123
Q

The rectal past medical history of all patients should include inquiry about
a. bowel habits.
b. dietary habits.
c. hemorrhoid surgery.
d. laxative use.

A

ANS: C
Past medical history should include inquiry about hemorrhoids, spinal cord injury, benign prostatic hypertrophy (BPH), prostate, colorectal, breast, ovarian, or endometrial cancers, and episiotomies of fourth-degree lacerations during delivery. Habits are part of the personal and social history; the use of laxatives is part of the history of the present illness

How well did you know this?
1
Not at all
2
3
4
5
Perfectly
124
Q

The effects of aging on the gastrointestinal system leads to more frequent experiences of
a. constipation.
b. prolonged satiety.
c. diarrhea.
d. prostate glandular atrophy.

A

ANS: A
Older adults experience an elevated pressure threshold for the sensation of rectal distention and are therefore susceptible to constipation. They also experience early satiety, fecal incontinence, and prostate glandular hypertrophy.

How well did you know this?
1
Not at all
2
3
4
5
Perfectly
125
Q

Factors associated with increased risk of prostate cancer include
a. African descent.
b. cigarette smoking.
c. a low-fat diet.
d. alcoholism.

A

ANS: A
The incidence rate of prostate cancer is 50% higher for African American men compared with white American men. African American men also have a higher mortality rate.

How well did you know this?
1
Not at all
2
3
4
5
Perfectly
126
Q

The caliber of the urinary stream is routine information in the history of
a. adolescents.
b. infants.
c. older adults.
d. sexually active young men

A

ANS: C
Routine questions about the caliber of the urinary stream and dribbling are directed toward older men because hypertrophy of the prostate gradually impedes urine flow.

127
Q

Equipment for examination of the anus, rectum, and prostate routinely includes gloves and
a. a hand mirror and gauze.
b. a lubricant and penlight.
c. slides and normal saline.
d. swabs and culture medium

A

ANS: B
Equipment for the examination includes a penlight, lubricating jelly, gloves, and fecal occult blood testing materials.

128
Q

Which is a risk factor for colorectal cancer?
a. High-fiber diet
b. Diet low in animal fats and proteins
c. Irish descen
d. Inherited BRAC2 mutation

A

ANS: D
History of intestinal polyps is considered a risk factor for colorectal cancer, as are diets low in fiber and high in animal fats and an ethnic background of Ashkenazi Jewish descent.

129
Q

Nodules found in the peritoneum through the anterior rectal wall
a. are found with bi-digital palpation.
b. are called shelf lesions.
c. are chronic fibrosis.
d. are found by having the patient bear down.

A

ANS: B
Because the anterior rectal wall is in contact with the peritoneum, you may be able to detect the tenderness of peritoneal inflammation and the nodularity of peritoneal metastasis. The nodules called shelf lesions are palpable just above the prostate in males and in the cul-de-sac of females.

130
Q

Perianal abscesses, fissures, or pilonidal cysts will cause the patient to experience
a. bulging and wrinkling.
b. constipation and pallor.
c. diarrhea and redness.
d. tenderness and inflammation.

A

ANS: D
Tenderness and inflammation to the perianal area may be related to an abscess, fistula, fissure, pilonidal cyst, or pruritus ani

131
Q

Palpation of the anal ring is done by
a. Bi-digital palpation with the thumbs.
b. inserting the smallest finger into the anus.
c. pressing a gauze pad over the anus.
d. rotation of the forefinger inside the anus

A

ANS: D
The anal muscular ring is palpated by rotating the examination finge

132
Q

The posterior surface of the prostate can be located by palpation of the
a. anal canal and perineum.
b. anterior wall of the rectum.
c. lateral wall of the anus.
d. lower abdomen and perineum.

A

ANS: B
Palpation of the rectal anterior wall facilitates posterior prostate location.

133
Q

The cervix may be palpated through the
a. anterior rectal wall.
b. internal umbilical wall.
c. lateral urethral meatus.
d. posterior uterine surface.

A

ANS: A
In women, the cervix can be palpated through the anterior rectal wall. It feels like a small, round mass

134
Q

Your patient’s chief complaint is repeated, pencil-like stools. Further examination should include
a. a stool culture.
b. parasite testing.
c. a digital rectal examination (DRE).
d. a prostate examination.

A

ANS: C
Persistent pencil-shaped stools are indicative of stenosis from scarring or pressure from a mass. DRE should be performed to assess for a mass

135
Q

Very light tan or gray stools may indicate
a. upper gastrointestinal bleeding.
b. obstructive jaundice.
c. lower gastrointestinal bleeding.
d. polyposis

A

ANS: B
Very light tan or gray stools suggest obstructive jaundice.

136
Q

tarry black stools should increase suspicion for
a. internal hemorrhoids.
b. rectal fistula.
c. upper intestinal tract bleeding.
d. prostatic cancer.

A

ANS: C
Upper intestinal tract bleeding results in tarry black stools

137
Q

Prostate-specific antigen (PSA) screening is controversial because
a. there are many false-negative results.
b. PSA is produced by many other tissues.
c. it is less sensitive than digital rectal examination.
d. no data have proved that it decreases mortality

A

ANS: D
There are no data confirming that PSA screening decreases mortality from prostate cancer.

138
Q

A lower spinal cord lesion may be indicated by which finding?
a. Lack of an anal wink
b. Rectal prolapse
c. Anal fistula
d. Small flaps of anal skin

A

ANS: A
Lightly touching the anal opening of an infant should produce a contraction referred to as an anal wink. A negative wink may indicate a lower spinal cord lesion.

139
Q

Pinworms and Candida may both cause
a. constipation.
b. hemorrhoids.
c. perirectal irritation.
d. perirectal protrusion

A

c

Pinworms and Candida both cause perirectal irritation and itching.

140
Q

Thrombosed hemorrhoids are
a. flabby skin sacs.
b. red, inflamed, and painful.
c. fluctuant soft papules.
d. blue, shiny, painful masses.

A

ANS: D
Thrombosed hemorrhoids appear as blue shiny masses at the anus; they contain clotted blood and are edematous and painful.

141
Q

Palpation of a normal prostate in an older adult is likely to feel
a. cool.
b. grainy.
c. polypoid.
d. rubbery.

A

ANS: D
Older men are more likely to experience prostate hypertrophy, which when palpated feels smooth, rubbery, and symmetric.

142
Q

Mr. Dunn is a 62-year-old man who has presented for a routine annual examination. On examination of the prostate you note a hard, irregular, painless nodule and obliteration of the median sulcus. These are signs of
a. benign prostatic hypertrophy.
b. cancer of the prostate.
c. long-standing prostatitis.
d. swelling as a result of aging.

A

ANS: B
Obliteration of the median sulcus is consistent with organ enlargement; however, the findings of a hard, irregular, and painless nodule indicate a cancerous growth

143
Q

The mother brings her 4-year-old to the clinic because the child complains of perianal itching. As part of your examination you complete a cellulose tape test. The cellulose tape test is used
for the detection of
a. enterobiasis.
b. carcinoma.
c. amebiasis.
d. steatorrhea.

A

ANS: A
Enterobiasis (pinworm infestation) is detected by the cellulose tape test. Pinworms are collected by applying tape to the perianal folds and then pressing the tape on a glass slide.

144
Q

. Mr. Sweeney is a 58-year-old man who has presented for a routine annual prostate examination. On examination, you note a normal prostate gland. Which of the following characteristics should describe the normal prostate? (Select all that apply.)
a. Rubbery consistency
b. About 4 cm in diameter
c. Fluctuant softness
d. Gland protruding 1 cm into the rectum
e. Firm, smooth, and slightly movable

A

ANS: B, D, E
The gland should feel like a pencil eraser, firm, smooth, and slightly movable and should be nontender. It has a diameter of 4 cm, with a 1-cm protrusion into the rectum.

145
Q

The first heart sound (S1) occurs because of the closure of the:
A. Aortic and mitral valves
B. Mitral and tricuspid valves
C. Pulmonic valve
D. Aortic valve

A

ANS: B
S1 is the closing of the mitral valve and with the tricuspid (T1) valve; together they are known as the atrioventricular (AV) valves. S2 is the closing of the aortic (A2) and pulmonic (P2) valves; together they are known as the semilunar valves.

146
Q

The second heart sound (S2) occurs because of the closure of the:
A. Aortic valve
B. Tricuspid valve
C. Aortic and pulmonic valve
D. A and B

A

ANS: C
S1 is the closing of the mitral valve and with the tricuspid (T1) valve; together they are known as the atrioventricular (AV) valves. S2 is the closing of the aortic (A2) and pulmonic (P2) valves; together they are known as the semilunar valves.

147
Q

To distinguish a physiologic split S2 heart sound, it is best to listen with the stethoscope at:
A. Left fifth intercostal space midclavicular line
B. Fourth intercostal space left sternal border with patient holding his or her breath
C. Second intercostal space left sternal border with inspiration
D. Fourth intercostal space right sternal border with expiratio

A

ANS: C
Normally, the S1 and S2 occur as single sounds. In some conditions, these sounds may be split and occur as two sounds. In healthy young adults, a physiologic split of S2 may be detected in the
second and third left interspaces during inspiration as a result of changes in the amount of blood returned to the right and left sides of the heart. During inspiration, there is an increased filling time
and therefore increased stroke volume of the right ventricle, which can delay closure of the pulmonic valve, causing the second heart sound to be split. This physiologic split differs from
other splits that are pathologic in origin in that it occurs with inspiration and disappears with expiration.

148
Q

An S3 gallop is commonly heard in:
A. Children with fever
B. Adults with heart failure
C. Children with aortic stenosis
D. Adults with hypertension

A

ANS: B
Pathologic S3, also called a ventricular gallop, is heard in adults and is associated with decreased myocardial contractility, HF, and volume overload conditions, as can occur with mitral or tricuspid
regurgitation. The sound is the same as a physiologic S3 and is heard just after S2 with the patient supine or in the left lateral recumbent position. The sound is very soft and can be difficult to hear.

149
Q

An S4 sound is commonly heard in:
A. Children with fever
B. Adults with atrial fibrillation
C. Adults with hypertension
D. Children with pulmonic stenosis

A

ANS: C
S4, also called an atrial gallop, occasionally occurs in a normal adult or well-trained athlete but is usually due to increased resistance to filling of the ventricle. Possible causes of a left-sided S4
include HTN, CVD, cardiomyopathy, and aortic stenosis. Possible causes of a right-sided S4 include pulmonic stenosis and pulmonary HTN. S4 is heard just before S1 with the patient supine or
in the left lateral recumbent position. The sound can be as loud as S1 and S2. S4 is not heard in patients with chronic atrial fibrillation due to no distinct atrial kick

150
Q

Which of the following heart sounds is commonly heard after myocardial infarction?
A. Friction rub
B. S4
C. S3
D. Opening snap

A

ANS: A
Opening snap is caused by the opening of a stenotic mitral or tricuspid valve and is heard early in
diastole along the lower left sternal border. It is high pitched and heard best with the diaphragm of the stethoscope. Friction rubs occur frequently after a myocardial infarction (MI) or with
pericarditis. The sound is a high-pitched grating, scratching sound—resulting from inflammation of the pericardial sac—that issues from the parietal and visceral surfaces of the inflamed pericardium as they rub together.

151
Q

Which of the following is the most important question to ask during cardiovascular health history?
A. Number of offspring
B. Last physical examination
C. Sudden death of a family member
D. Use of caffeine

A

ANS: C
The sudden death of a family member is an important question to ask in the health history because it reveals the cardiovascular disease risk of the patient. Sudden death is usually due to an acute
cardiovascular event, such as myocardial infarction, cardiac dysrhythmia, or stroke.

152
Q

Cardiovascular disease risk increase in women after age:
A. 30
B. 40
C. 45
D. 55

A

ANS: D
In men, cardiovascular risk increases after age 45; however, in women, the risk of cardiovascular disease increased after age 55. Estrogen is cardioprotective and so women are at highest risk for
cardiovascular disease after menopause. The average age of natural menopause for women is age 55.

153
Q

Which of the following blood pressure measurements is categorized as prehypertension?
A. 110/78
B. 129/85
C. 142/80
D. 145/92

A

ANS: B
Ideal blood pressure is less than 120/80, pre-hypertension is regarded as 120 to 139 systolic / 80 to 89, and hypertension is greater than or equal to 140/90

154
Q

Xanthelasma is a skin condition that should alert the clinician to in a patient.
A. Familial hyperlipidemia
B. Type 2 diabetes
C. Congenital heart disease
D. Peripheral arterial disease

A

ANS: A
When conducting the inspection part of a physical examination, inspect the skin around the eyes for xanthelasma. Xanthelasma is a skin condition that occurs in familial hyperlipidemia that causes
high cholesterol levels and early cardiovascular disease in patients. Small, yellow fatty deposits are located around the eyes.

155
Q

When palpating the chest, you find the point of maximal impulse (PMI) in the left mid-axillary region. This can be indicative of:
A. Normal PMI
B. Congenital heart disease
C. Ventricular hypertrophy
D. Hypertension

A

ANS: C
The examiner should palpate the point of maximal impulse (PMI) and the precordium for heaves or lifts, seen in ventricular hypertrophy. The apical impulse is easily observed in the pediatric
client but not always visible in the adult. An accentuated or displaced apical impulse may indicate ventricular hypertrophy.

156
Q

Upon inspecting the patient, you find jugular venous distension. This is a sign of:
A. Left ventricular hypertrophy
B. Right ventricular failure
C. Hypertension
D. Valve disease

A

ANS: B
In right ventricular failure, hydrostatic pressure builds up back into the right atrium and superior as well as inferior vena cava. Venous congestion occurs throughout the body. A sign of venous
congestion of the superior vena cava is jugular vein distension. Signs of right ventricular failure include jugular venous distension, ascites, hepatomegaly, splenomegaly, and ankle edema

157
Q

On an electrocardiogram, you see a deepened Q wave that is greater than one-third the height of the QRS complex. This is indicative of:
A. Acute myocardial infarction
B. Acute myocardial ischemia
C. Left ventricular hypertrophy
D. Past myocardial infarction

A

ANS: D
A pathologic Q wave, indicative of a past myocardial infarction, measures greater than 0.04 seconds and is greater than one-third the height of the QRS complex

158
Q

Your 35-year-old female patient complains of feeling palpitations on occasion. The clinician should recognize that palpitations are often a sign of:
A. Anemia
B. Anxiety
C. Hyperthyroidism
D. All of the above

A

ANS: D
Palpitations sensed by the patient are commonly indicative of atrial fibrillation, tachycardia, or premature ventricular contractions. Anemia, anxiety, and hyperthyroidism are all possible
etiologies of these cardiac rhythm disturbances.

159
Q

A pulse rate of 56 beats per minute can be normal in:
A. Elderly patients
B. Newborns
C. Athletic individuals
D. Hypertensive patients

A

ANS: C
Athletic individuals commonly have a low pulse rate because their heart is well-conditioned and physiologically more efficient. A strong heart has physiologically enlarged muscle chambers which
contract slowly and efficiently deliver an adequate blood volume to the body with each contraction. In an elderly or untrained individual, bradycardia is more concerning. Although normal aging includes a decrease in SA node and AV node conduction, thus increasing the
likelihood of bradycardia, sinus node pathology and heart blocks should be suspected in the elderly. Newborns have a normal pulse rate of 120 beats per minute. If the patient has hypertension, this does not decrease the heart rate.

160
Q

Your patient has a dysrhythmia and has been on a diuretic for 2 months. Which of the following should be suspected?
A. Potassium imbalance
B. Sodium deficit
C. Calcium imbalance
D. Insufficient diuretic

A

ANS: A
The most common electrolyte disturbance in patients on diuretics is hypokalemia. Hypokalemia is a common cause of cardiac rhythm disturbance

161
Q

Your patient has suffered an inferior wall myocardial infarction. This is most commonly due to an obstruction in the:

A. Posterior branch of the right coronary artery
B. Circumflex branch of the left coronary artery
C. Right main coronary artery
D. Left main coronary artery

A

ANS: A
Blockage in the right coronary artery (RCA) results in damage to the posterior/inferior area of the heart. The left main coronary artery (LCA) branches off to the left anterior descending artery
(LAD) and the left circumflex artery (LCX). A highly stenotic LCA or proximal LAD can cause significant heart damage and is often termed the “widow maker.” Blockage in the LAD results in
damage to the anterior portion of the heart. Blockage in the circumflex branch artery (CFX) results in damage to the posterior and lateral areas

162
Q

Cardiac chest pain is most often described as:
A. Stabbing, piercing pain
B. Pain with inhalation
C. Crushing, squeezing pain
D. Burning, gnawing pain

A

ANS: C
Typical characteristics that indicate acute coronary syndrome (ACS) include crushing, squeezing substernal chest pain with radiation to the neck or left arm, a score of greater than 7 on the pain
scale, an association with exertion or stress with relief on rest, a duration of minutes, and associated symptoms of nausea, diaphoresis, weakness, or shortness of breath.

163
Q

On an electrocardiogram (ECG), an anterior wall myocardial infarction is demonstrated on leads:
A. II, III, AVR
B. II, III, AVF
C. V1, V2, V3, V4
D. I, AVL, V5, V6

A

ANS: C
With angina and MI, acute ischemic changes may be seen on the 12-lead EKG. Each lead reflects an area of the heart, and an EKG can determine the location of the ischemia. The lateral wall of the
heart is reflected in leads I, aVL, V5, and V6. The inferior wall is reflected in leads II, III, and a VF. The anterior wall is reflected in leads V1, V2, V3, and V4. The posterior wall is reflected in leads V1,
V2, and V3. With ischemia, an EKG changes need to occur in two contiguous leads. Reciprocal EKG changes can be seen in the area of the heart opposite the injured area.

164
Q

Which is the most specific and sensitive test for validating a myocardial infarction?
A. 12-lead EKG
B. Troponin
C. CK-MB
D. CT scan

A

ANS: B
Diagnostic studies of myocardial infarction include:
* CK-MB—The serum level of CK-MB is elevated above normal in the first few hours after MI
and returns to normal within 72 hours. The levels can also be elevated following trauma or with
progressive muscular dystrophy.
* Troponin—An inhibitory protein found in muscle fibers, troponin is elevated within 4 hours of
an MI and stays elevated for 7 to 10 days. It is more sensitive and specific than creatine kinase for
cardiac muscle but may be falsely elevated in patients with kidney dysfunction.
* 12-lead EKG—The practitioner should look for signs of acute ischemia, such as ST-segment
elevation or depression, arrhythmias, and conduction delays. An EKG is minimally helpful in
diagnosing pericarditis except in the case of cardiac tamponade or constrictive pericarditis where
decreased amplitude may be seen.
* Imaging—Studies such as computed tomography (CT) scan, electron beam computed
tomography (EBCT) scan, positron emission tomography (PET) scan, magnetic resonance imaging
(MRI), and single photon emission computed tomography (SPECT) scan can assist in diagnosing
CAD, aortic aneurysms, cardiac masses, myocardial disease, and pericardial disease.

165
Q

It is important for clinicians to recognize that individuals with often sustain silent myocardial infarction.

A. Diabetes mellitus
B. Hypertension
C. Valvular disorders
D. Congenital heart defects

A

ANS: A
Pain sensitivity may be blunted in persons with diabetes. Peripheral and autonomic neuropathy is common in long-term diabetes. Individuals with diabetes often suffer infection or other disorders
without sensing the pain and sustain complications without warning.

166
Q

The pain associated with pericarditis is .
A. Crushing and squeezing
B. Constant
C. Worse with inspiration
D. Only present with fever

A

c

Unlike the symptoms associated with ACS, the pain associated with pericarditis is sharp and stabbing; it may worsen with inspiration or when lying flat or leaning forward. Associated symptoms may include shortness of breath, fever, chills, and malaise.

167
Q

Pain associated with a dissecting thoracic aortic aneurysm is commonly described as:
A. Retrosternal crushing and squeezing
B. Chest stabbing and sharp
C. Ripping and tearing in the chest or thoracic back
D. Worse with inspiration

A

ANS: C
Aortic aneurysms are often asymptomatic. However, in a dissecting aortic aneurysm , symptoms are often described as tearing or ripping in the chest, back, or abdomen.

168
Q

The pain of can frequently be mistaken for cardiac chest pain.
A. Gastroesophageal reflux disease (GERD)
B. Peptic ulcer disease (PUD)
C. Cholecystitis
D. All of the above

A

ANS: D
It is often difficult to differentiate the symptoms of gastroesophageal reflux disease (GERD) or peptic ulcer disease (PUD) from cardiac symptoms. A thorough history and diagnostic tests are
necessary. Patients with a history of GERD or PUD should still be worked up for a cardiac etiology, particularly if the characteristics of the symptoms or the history have changed to raise the
index of suspicion for cardiac disease. The pain of cholecystitis, also sometimes mistaken for cardiac pain, typically presents with right upper quadrant pain with radiation to the thoracic region
of the back.

169
Q

The pain of pancreatitis is described as:
A. Abdominal sharp and piercing pain in the left upper quadrant
B. Dull and cramping pain in the right upper quadrant
C. Severe, epigastric pain radiating straight into the back
D. Sharp pain radiating to the shoulder

A

ANS: C
The pain of pancreatitis is severe, steady, and “boring”—radiating from the epigastric region through to the back. It is often accompanied by nausea and vomiting, tachycardia, hypotension,
and diaphoresis. These symptoms are also seen in MI; however, the exquisite abdominal tenderness present in pancreatitis assists in differentiating it from cardiac pain

170
Q

The pain of costochondritis typically .
A. Mimics cardiac crushing and squeezing pain
B. Worsens with movement and full inspiration
C. Radiates from epigastrium into the back
D. Is a tearing and ripping pain

A

ANS: B
Costochondritis, which is inflammatory pain of the chest wall, can often be differentiated from cardiac pain through history. A history of injury, heavy lifting, contact sports, excessive coughing,
or late-stage pregnancy (which stretches the intercostal muscles) leads the examiner to consider chest wall pain. This often occurs in a younger population with no cardiac risk factors. One of the
most helpful differentiating symptoms is that the pain is increased with movement, cough, or, in some cases, respiration.

171
Q

The medical record of your patient lists a grade III systolic murmur. This indicates the patient has a heart murmur that is:
A. Soft and after S2
B. Loud and crescendo in quality
C. Moderately loud and after S1
D. Loud and after S2

A

ANS: C
A grade III murmur is moderately loud and a systolic murmur immediately follows S1

172
Q

The radiation of a mitral valve murmur is commonly heard in the:
A. Carotid arteries
B. Left mid-axillary line
C. Base of the heart
D. Left mid-clavicular line

A

b

A thorough cardiac examination is performed with the patient sitting, leaning forward, lying, and in the left lateral recumbent position. Some murmurs are heard better in different positions. Listen
over the carotids for radiation of an aortic or pulmonic murmur, in the left mid-axillary line for radiation of a mitral murmur, and in the epigastric area for a bruit, indicating an aneurysm.

173
Q

The murmur of aortic stenosis is best heard in the:
A. Left second intercostal space left sternal border
B. Left fifth intercostal space mid-clavicular line
C. Right fourth intercostal space right sternal border
D. Right second intercostal space right sternal border

A

. ANS: D
Aortic stenosis is heard best in the second right intercostal space with the client leaning forward. The murmur is harsh, loud, and often associated with a thrill. It may radiate to the neck, left sternal
border, and, in some cases, to the apex.

174
Q

The pulmonary valve is best heard over the:
A. left second intercostal space left sternal border
B. left fifth intercostal space mid-clavicular line
C. right fourth intercostal space right sternal border
D. right second intercostal space right sternal border

A

ANS: A
The valves are best heard over the chest at specific areas. The aortic valve is best heard over the
second intercostal space right sternal border and the pulmonic valve is best heard over the second
intercostal space left sternal border. The mitral valve is best heard over the fifth intercostal space
mid-clavicular line.

175
Q

The key sign(s) of aortic stenosis are:
A. Syncope
B. Dyspnea
C. Angina
D. All of the above

A

ANS: D
Syncope, angina, and dyspnea (remembered with the acronym SAD) on exertion are the classic symptoms of aortic stenosis. If syncope occurs with exertion, the aortic stenosis is typically severe.
Angina may be present because of decreased perfusion of the left ventricle due to LVH rather than CAD, but both exist in many cases.

176
Q

Classically in mitral valve prolapse, the clinician can hear a(n) .

A. Mid-systolic click followed by a grade I murmur that crescendos up to S2
B. Opening snap followed by a grade III holosystolic murmur
C. Crescendo-decrescendo grade I diastolic murmur after S2
D. Rough grade III holosystolic murmur that obscures S1 and S2

A

ANS: A
In mitral valve prolapse (MVP), a portion of the mitral valve flops open up into the left atrium, giving rise to a classic mid-systolic click followed by a soft grade I murmur that crescendos up to
S2. It is high pitched and is heard best at the apex or left sternal border.

177
Q

The best diagnostic test that allows analysis of a heart murmur is:
A. CT scan
B. Echocardiogram
C. MRI
D. ECG

A

ANS: B
Echocardiography is the best diagnostic test for assessing a heart murmur. In addition to visualizing the diseased valve, echocardiography can assist in determining the size of the heart
chambers involved in the heart murmur in real time.

178
Q

The most common cause of tricuspid regurgitation is:
A. Left ventricular hypertrophy
B. Left atrial enlargement
C. Aortic stenosis
D. Pulmonary hypertension

A

d

The murmur of tricuspid regurgitation is heard best at the left sternal border and may radiate to the right of the sternum. It is pansystolic, high pitched, and blowing and increases with respiration.
Tricuspid regurgitation may be associated with right ventricular hypertrophy resulting in a right parasternal lift. When right ventricular failure occurs, jugular venous distention occurs with a
prominent v wave, and liver enlargement may be present. There may be secondary right atrial enlargement owing to backflow into the right atrium. The most common initiator is pulmonary
HTN

179
Q

The tricuspid valve is best heard over the:
A. Third intercostal space left sternal border
B. Fifth intercostal space right sternal border
C. Fourth intercostal space left sternal border
D. Third intercostal space right sternal border

A

ANS: C
The tricuspid valve is best heard over the fourth intercostal space left sternal border. Erb’s point is located over the third intercostal space left sternal border. The mitral valve is best heard over the
fifth intercostal space in the mid-clavicular line. The aortic valve is best heard over the second intercostal space right sternal border. The pulmonic valve is best heard over the second intercostal
space left sternal border.

180
Q

From Erb’s point, all the heart valves can be heard equally. Erb’s point is located over the:

A. Third intercostal space left sternal border
B. Fifth intercostal space right sternal border
C. Fourth intercostal space left sternal border
D. Third intercostal space right sternal border

A

ANS: A
The tricuspid valve is best heard over the fourth intercostal space left sternal border. Erb’s point is where all valves can be heard equally well. Erb’s point is located over the third intercostal space
left sternal border. The mitral valve is best heard over the fifth intercostal space in the midclavicular line. The aortic valve is best heard over the second intercostal space right sternal border.
The pulmonic valve is best heard over the second intercostal space left sternal border.

181
Q

Upon examination of a child, an innocent systolic murmur is heard at the second intercostal space left sternal border. This is usually due to:
A. Atrial septal defect
B. Patent foramen ovale
C. Low flow velocity
D. High flow turbulence

A

ANS: D
An innocent murmur is a type of systolic murmur that results from turbulent blood flow and is not associated with heart disease. Innocent murmurs occur commonly in children and young adults andreflect the contractile force of the heart, resulting in greater velocity of flow during early systole. they are heard best in the second and third left interspaces along the left sternal border or at the apex

182
Q

In mitral stenosis, the murmur occurs:

A. From S1 through S2 as a holosystolic murmur
B. After an opening snap that is heard after S2 during diastole
C. As a soft mid-systolic click
D. A loud crescendo-decrescendo systolic murmur

A

ANS: B
Mitral stenosis results from thickening and stiffening of the mitral valve, usually secondary to rheumatic fever. The murmur is generally grade I to IV and low pitched; therefore, it is heard
better with the bell at the apex in the left lateral recumbent position. The first heart sound (S1) is loud, followed by S2 and a loud opening snap that precedes the murmur.

183
Q

In mitral stenosis, the murmur can be best heard with the patient in the:
A. Squatting position
B. Seated position
C. Left lateral recumbent
D. Supine position

A

c

Mitral stenosis results from thickening and stiffening of the mitral valve, usually secondary to rheumatic fever. The murmur is generally grade I to IV and low pitched; therefore, it is heard better with the bell at the apex in the left lateral recumbent position. The first heart sound (S1) is loud, followed by S2 and a loud opening snap that precedes the murmur.

184
Q

Due to increased left atrial pressure, a patient with mitral stenosis often suffers from:
A. Pulmonary congestion
B. Hepatomegaly
C. Jugular venous distension
D. Ventricular tachycardia

A

ANS: A
In mitral stenosis, the most common presenting symptoms are dyspnea on exertion and hemoptysis due to pulmonary congestion. The pulmonary congestion is caused by increased left atrial pressure
related to the decrease in left atrial emptying. Crackles may be heard at the lung bases but are not present in all patients with pulmonary congestion. Orthopnea may be present because the lungs
become more congested in the recumbent position. In addition, atrial fibrillation often develops in patients with mitral stenosis, which, in turn, worsens the pulmonary congestion.

185
Q

In aortic stenosis, the patient’s point of maximal impulse is commonly located at the:
A. Fifth intercostal space mid-clavicular line
B. Fifth intercostal space mid-axillary line
C. Second intercostal space left sternal border
D. Second intercostal space right sternal border

A

ANS: B
In aortic stenosis, the left ventricle has to work against high resistance and eventually, left ventricular hypertrophy (LVH) develops. The LVH causes an enlarged left ventricle, and the point
of maximal impulse located at the fifth intercostal space moves leftward toward the mid-axillary line.

186
Q

Aortic regurgitation occurs after S2 during because there is turbulent flow that refluxes into the left ventricle after the aortic valve closes.
A. Early diastole
B. Late diastole
C. Early systole
D. Mid-systole

A

ANS: A
The aortic valve closes creating S2. After the aortic valve closes, in aortic regurgitation, some blood flows back into the left ventricle. S1 through S2 is the systolic time frame. S2 through S1 is
the diastolic time frame. The sound of a leaky aortic valve would be heard after the aortic valve closes (S2) during early diastole.

187
Q

The murmur of a ventricular septal defect (VSD) occurs when the ventricle contracts and blood flows from the left ventricle into the right ventricle. This creates a heart murmur.
A. Holodiastolic
B. Early diastolic
C. Holosystolic
D. Late systolic

A

ANS: C
Ventricular septal defect (VSD) is a congenital heart defect in which oxygenated blood is shunted from a higher-pressured left ventricle to a lower-pressured right ventricle through an abnormal
opening in the ventricular septum. This left-to-right shunt causes an increased blood flow across the pulmonic valve. The signs and symptoms depend on the size of the defect and the age of the
patient. A VSD murmur occurs after the mitral valve closes (S1). Characteristic of a VSD is a loud, harsh, pansystolic murmur at the lower left sternal border, usually accompanied by a thrill.

188
Q

An atrial septal defect (ASD) causes a left to right shunt, which enlarges the right atrium. Because of this effect, which of the following conditions often occur with ASD?
A. Asthma
B. Jugular venous distension
C. Atrial fibrillation
D. B & C

A

ANS: D
ASD is a congenital abnormality in which oxygenated blood is shunted from a higher-pressured
left atrium to a lower-pressured right atrium through an abnormal opening in the atrial septum.
Atrial arrhythmias, especially atrial fibrillation, are common in the adult population with ASD.
ASDs are often accompanied by other congenital heart defects, but in an uncomplicated lesion,
patients are often asymptomatic until early adulthood, when they present with dyspnea on exertion
or palpitations resulting from atrial arrhythmia. Because patients may be asymptomatic for many
years, right HF can be the first sign, and patients may present with jugular venous distension,
edema, and ascites

189
Q

A patient with hypertension who has hyperlipidemia should aim for LDL measurement to be:
A. 130 mg/dL or less
B. 40 mg/dL or less
C. 100 mg/dL or less
D. 60mg/dL or less

A

ANS: C
The aim for primary prevention is to keep LDL levels below 130 mg/dL and HDL levels above 40
mg/dL. Although primary prevention lowers a person’s risk of heart disease and MI, it has shown
only small, if any, effect on all-cause mortality. In patients with known CVD or diabetes, the target
cholesterol levels are more stringent, aiming for an LDL level below 100 mg/dL and an HDL level
above 60 mg/dL.

190
Q

Your 47-year-old female patient has a waist to hip ratio of 1. In terms of cardiovascular disease risk, this is considered:
A. Ideal
B. Greater than acceptable limits
C. Less than acceptable limits
D. Within acceptable limits

A

ANS: B
The physical examination includes measurement of height and weight to calculate body mass
index (BMI). The formula for calculating BMI is wt(kg)/ht(m2
). A waist/hip ratio is also an
indicator for risk of heart disease. A ratio greater than 0.85 for women and greater than 0.95 for
men is considered to place individuals at increased risk, especially if accompanied by
hyperinsulinemia or diabetes. These are part of a constellation of symptoms termed metabolic
syndrome that indicates the greatest risk for the development of heart disease.

191
Q

**

Which of the following conditions is/are part of metabolic syndrome?
A. Hypertension
B. Hyperlipidemia
C. Insulin resistance
D. All of the above

A

ANS: D
Characteristics of Metabolic Syndrome

Hypertension
* Dyslipidemia
* Central obesity
* Glucose intolerance with hyperinsulinemia (insulin resistance)

192
Q

The target body mass index for women is:
A. 27 kg/m2
B. 25 kg/m2
C. 22 kg/m2
D. 16 kg/m2

A

ANS: C
Weight loss through diet and exercise has been found to be the most important factor in the prevention of the progression of metabolic syndrome. Insulin sensitivity increases with weight loss
and is thought to be due to the loss of visceral fat. The target BMI is less than 22 kg/m2 for women and less than 27 kg/m2 for men. BMI of 16 kg/m2 is considered too low

193
Q

Dyspnea, cough, and pulmonary crackles are symptoms that can occur in left ventricular failure and respiratory disorders, such as pneumonia. Which of the blood tests below can be used to
differentiate cardiovascular from pulmonary disease?
A. B type natriuretic peptide (BNP)
B. Pulse oximetry
C. Arterial blood gases
D. High sensitivity C reactive protein (hs-CRP)

A

ANS: A

Brain natriuretic peptide (BNP)—For patients with cardiovascular disease and comorbid
respiratory disease, BNP can help differentiate a cardiac from a respiratory etiology of dyspnea.
High levels of BNP in the blood, greater than 400 mg/dL, occur if heart failure is present.

194
Q

When an examiner presses on the liver and elicits hepato-jugular reflux, which of the following conditions is likely?
A. Left ventricular failure
B. Right ventricular failure
C. Hepatomegaly
D. Pulmonary edema

A

ANS: B
In right ventricular failure, the abdomen should be examined particularly for right upper quadrant
discomfort related to hepatic congestion and enlargement. The examiner can also check for hepatic
jugular reflux by placing sustained pressure on the liver while observing for jugular venous
distention. In right HF, ascites may also be present. Examine the extremities for edema seen in
right HF

195
Q

Your patient has had hypertension for 10 years, a myocardial infarction 5 years ago, and now complains of dyspnea on exertion, cough, and 3-pillow orthopnea. Which of the following conditions is likely?
A. Right ventricular failure
B. Pulmonary embolism
C. Cor pulmonale
D. Left ventricular failure

A

ANS: D
Left ventricular failure is most commonly characterized by dyspnea on exertion, cough, fatigue,
orthopnea, PND, cardiac enlargement, crackles, gallop rhythm, and pulmonary congestion. Right
ventricular failure is commonly characterized by dependent edema, elevated venous pressure,
hepatomegaly, and possibly ascites. Pulmonary embolism presents with shortness of breath and
tachycardia. Cor pulmonale is a heart disorder that occurs because of lung disease. The lungs are
dysfunctioning which causes hypoxia and reflex pulmonary artery vasoconstriction. The right
ventricle has to eject blood into the pulmonary artery, however, there is high afterload to deal
because of pulmonary vasoconstriction. The high workload against the right ventricle eventually
causes the right ventricle to fail. This right ventricular failure because of lung disease is called cor
pulmonale

196
Q

Your patient complains of worsening ankle edema and weight gain over the last week. On physical examination, you note jugular venous distension, ascites, hepatomegaly, and splenomegaly. These
conditions are indicative of:

A. Left ventricular failure
B. Pulmonary embolism
C. Right ventricular failure
D. Myocardial infarction

A

ANS: C
Right ventricular failure is characterized by dependent edema, elevated venous pressure,
hepatomegaly, and possibly ascites. Pulmonary embolism presents with sudden shortness of breath
and tachycardia. Left ventricular failure is characterized by dyspnea on exertion, cough, fatigue,
orthopnea, PND, cardiac enlargement, crackles, gallop rhythm, and pulmonary congestion.
Myocardial infarction presents with crushing retrosternal chest pain, pallor, dyspnea, and
diaphoresis.

197
Q

A 23-year-old patient presents the emergency department with high fever, chills, extreme fatigue, and arthralgias. Your physical examination reveals grade II heart murmur heard loudest over the
fourth intercostal space left sternal border. The arms of the patient reveal past intravenous drug abuse. The clinician should recognize these are signs and symptoms of:

A. Pulmonary embolism
B. Right ventricular failure
C. Functional heart murmur
D. Bacterial endocarditis

A

ANS: D
Bacterial endocarditis is a microbial infection of the endocardium. The most common causative
organisms are Staphylococcus aureus, group A streptococcus, pneumococcus, and gonococcus.
Risk factors include valvular disease, intravenous drug use, dental disease, and invasive diagnostic
procedures. Initially, the signs and symptoms are similar to those of other systemic illnesses,
including fever, chills, arthralgias, malaise, and fatigue. Petechiae, anemia, weight loss, new or
worsening heart murmur, and emboli alert the examiner to a more serious disease process.

198
Q

A 75-year-old patient complains of pain and paresthesias in the right foot that worsens with exercise and is relieved by rest. On physical examination you note pallor of the right foot, capillary refill of 4 seconds in the right foot, +1 dorsalis pedis pulse in the right foot, and +2 pulse in left foot. Which of the following is a likely cause of the signs and symptoms?

A. Arterial insufficiency
B. Femoral vein thrombus
C. Venous insufficiency
D. Peripheral neuropathy

A

ANS: A
Intermittent claudication is pain in the leg or foot that becomes worse with exercise and is relieved
by rest. The classic signs of peripheral arterial disease include pain, pallor, weak pulse,
paresthesias, and palpable coolness. The signs of venous thrombosis are erythema, ropiness, as
well as warmth and tenderness along the course of the vein. Edema of the leg and Homan’s sign of
the foot are also common.

199
Q

Which of the following ankle-brachial index measurements require a referral to a vascular consultant?
A. ABI 1.2
B. ABI 1
C. ABI 0.9
D. ABI 0.5

A

ANS: D
The ankle-brachial index (ABI) is currently the easiest, least expensive noninvasive method for
diagnosing PAD and is particularly helpful in the office and home settings. The ABI is obtained by
the following steps:
1. Obtain brachial systolic pressure in both arms. Select the higher of these two values.
2. Use Doppler stethoscope to obtain systolic pressure in the dorsalis pedis or posterior tibialis
vessel.
3. Divide ankle pressure by the higher brachial pressure.
The index should be 1 or higher. If it is less than 0.5, impairment to blood flow is significant. An
abnormal ABI indicates the need for a vascular consult.

200
Q

Your patient complains of a feeling of heaviness in the lower legs daily. You note varicosities, edema, and dusky color of both ankles and feet. Which of the following is the most likely cause for
these symptoms?

A. Femoral vein thrombosis
B. Femoral artery thrombus
C. Venous insufficiency
D. Musculoskeletal injury

A

ANS: C
Chronic venous insufficiency can be a long-term complication of venous thrombosis, owing to the
destruction of valves in the deep veins. The calf muscle pump that returns blood from the lower
legs is damaged, increasing ambulatory pressure in the calf veins. A constellation of symptoms is
set up: aching or pain in the lower legs, edema, thinning and hyperpigmentation of the skin,
superficial varicosities, venous stasis, and ulceration. Ankle edema is often the earliest sign.

201
Q

After multiple pregnancies, the following vascular disorder is common:

A. Deep venous thrombosis
B. Varicose veins
C. Peripheral arterial disease
D. Aortic aneurysm

A

ANS: B
Often a precursor to chronic venous insufficiency, varicose veins are usually caused by
occupations that involve prolonged standing or sitting in one place, overweight, pregnancy, or a
familial tendency. They may increase the patient’s risk for DVT, or they may occur secondary to a
DVT. Blockage to lymphatic flow can cause varicosities as seen with pelvic neoplasm. They
appear as long, dilated, tortuous veins in the lower extremities.

202
Q

When performing abdominal assessment, the clinician should perform examination techniques in the following order:

A. Inspection, palpation, percussion, and auscultation
B. Inspection, percussion, palpation, and auscultation
C. Inspection, auscultation, percussion, and palpation
D. Auscultation, palpation, percussion, and inspection

A

ANS: C
The abdominal examination begins with inspection, followed by auscultation, percussion, and palpation. Light palpation should precede deep palpation. Auscultating before percussion or
palpation allows the examiner to listen to the abdominal sounds undisturbed. Moreover, if pain is present, it is best to leave palpation until last and to gather other data before possibly causing the
patient discomfort.

203
Q

The clinician should auscultate the abdomen to listen for possible bruits of the:
A. Aorta
B. Renal artery
C. Iliac artery
D. All of the above

A

ANS: D
Perform auscultation before palpation so as to hear unaltered bowel sounds. Listen for bruits over the aorta and the iliac, renal, and femoral arteries

204
Q

On abdominal examination, which of the following is assessed using percussion?
A. Liver
B. Kidneys
C. Pancreas
D. Esophagus

A

ANS: A
The purpose of liver percussion is to measure the liver size. The technique used to percuss the liver
is as follows:
1. Starting in the midclavicular line at about the 3rd intercostal space, lightly percuss and move
down.
2. Percuss inferiorly until dullness denotes the liver’s upper border (usually at fifth intercostal
space in MCL).
3. Resume percussion from below the umbilicus on the midclavicular line in an area of tympany.
4. Percuss superiorly until dullness indicates the liver’s inferior border.
5. Measure span in centimeters. Normal liver span: clinically estimated at midclavicular line: 6-12
cm and midsternal line: 4-8 cm.

205
Q

In abdominal assessment, a digital rectal examination is performed to assess for:
A. Hemorrhoids
B. Prostate size
C. Blood in stool
D. Ureteral stenosis

A

ANS: D
A digital rectal examination is included in the abdominal examination. Note skin changes or lesions in the perianal region or the presence of external hemorrhoids. Insert the gloved index
finger into the anus with the patient either leaning over or side-lying on the examination table, and note any internal hemorrhoids or fissures. Check the stool for occult blood. For males, the rectal
examination is necessary for direct examination of the prostate. Ureteral stenosis is detected by angiographt.

206
Q

Rebound tenderness of the abdomen is a sign of:
A. Constipation
B. Peritoneal inflammation
C. Elevated venous pressure
D. Peritoneal edema

A

b

Rebound tenderness is tested by slowly pressing over the abdomen with your fingertips, holding the position until pain subsides or the patient adjusts to the discomfort, and then quickly removing
the pressure. Rebound pain, a sign of peritoneal inflammation, is present if the patient experiences a sharp discomfort over the inflamed site when pressure is released

207
Q

While assessing the abdomen, the clinician deeply palpates the left lower quadrant of the abdomen, and this causes pain in the patient’s right lower abdomen. This is most commonly indicative of:
A. Constipation
B. Diverticulitis
C. Appendicitis
D. Hepatitis

A

ANS: C
Appendicitis is suggested by a positive Rovsing’s sign. This sign is positive when there is referred rebound pain in the right lower quadrant when the examiner presses deeply in the left lower
quadrant and then quickly releases the pressure.

208
Q

Your patient complains of severe right lower quadrant abdominal pain. To assess the patient for peritoneal inflammation, the examiner should:
A. Percuss the right lower quadrant of the abdomen
B. Deeply palpate the right lower quadrant of the abdomen
C. Auscultate the right lower quadrant for hyperactive bowel sounds
D. Strike the plantar surface of the patient’s heel while the patient is supine

A

ANS: D
Ask the patient to stand with straight legs and to raise up on toes. Then ask the patient to relax, allowing the heel to strike the floor, thus jarring the body. A positive heel strike is indicative of appendicitis and peritoneal irritation. Alternatively, strike the plantar surface of the heel with your fist while the patient rests supine on the examination table.

209
Q

Your patient is lying supine and you ask him to raise his leg while you place resistance against the thigh. The examiner is testing the patient for:
A. Psoas sign
B. Obturator sign
C. Rovsing’s sign
D. Murphys’sign

A

ANS: A
To examine the patient for appendicitis, the clinician can test the patient for psoas sign. This is done in the following manner: Place a hand on the patient’s thigh just above the knee and ask the
patient to raise the thigh against your hand. This contracts the psoas muscle and produces pain in patients with an inflamed appendix.

210
Q

A patient is lying supine and the clinician deeply palpates the right upper quadrant of the abdomen while the patient inhales. The examiner is testing the patient for:
A. Psoas sign
B. Obturator sign
C. Rovsing’s sign
D. Murphys’sign

A

ANS: D
Murphy’s Sign is elicited by deeply palpating the right upper quadrant of the abdomen. Pain is present on deep inspiration when an inflamed gallbladder is palpated by pressing the fingers under
the rib cage. Murphy’s sign is positive in cholecystitis.

211
Q

Your patient has abdominal pain, and it is worsened when the examiner rotates the patient’s right hip inward with the knee bent and the obturator internus muscle is stretched. This is a sign of:
A. Diverticulitis
B. Cholecystitis
C. Appendicitis
D. Mesenteric adenitis

A

ANS: C
A positive obturator sign indicates appendicitis. Pain is elicited by inward rotation of the right hip with the knee bent so that the obturator internus muscle is stretched.

212
Q

On abdominal examination as the clinician presses on the right upper quadrant to assess liver size, jugular vein distension becomes obvious. Hepatojugular reflux is indicative of:
A. Acute hepatitis
B. Right ventricular failure
C. Cholecystitis
D. Left ventricular failure

A

ANS: B
Hepatojugular reflux is elicited by applying firm, sustained hand pressure to the abdomen in the midepigastric region while the patient breathes regularly. Observe the neck for elevation of the
jugular venous pressure (JVP) with pressure of the hand and a sudden drop of the JVP when the hand pressure is released. Hepatojugular reflux is exaggerated in right heart failure

213
Q

Your patient demonstrates positive shifting dullness on percussion of the abdomen. This is indicative of:
A. Cholecystitis
B. Appendicitis
C. Ascites
D. Hepatitis

A

ANS: C
To assess the patient for ascites, test for shifting of the peritoneal fluid to the dependent side by rolling the patient side to side and percussing for dullness on the dependent side of the abdomen.

214
Q

Your 44-year-old female patient complains of right upper quadrant pain. Her skin and sclera are yellow, and she has hyperbilirubinemia and elevated liver enzymes. The clinician should suspect:
A. Acute pancreatitis
B. Biliary duct obstruction
C. Acute hepatitis
D. Atypical appendicitis

A

ANS: B
In cholecystitis, acute colicky pain is localized in the RUQ and is often accompanied by nausea and vomiting. Murphy’s sign is frequently present. Fever is low grade, and the increase in
neutrophilic leukocytes in the blood is slight. Acute cholecystitis improves in 2 to 3 days and resolves within a week; however, recurrences are common. If acute cholecystitis is accompanied
by jaundice and cholestasis (arrest of bile excretion), suspect common duct obstruction.

215
Q

The most common cause of acute pancreatitis is:
A. Trauma
B. Hepatitis virus A
C. Hyperlipidemia
D. Alcohol abuse

A

ANS: D
Biliary tract disease and alcoholism account for 80% or more of the pancreatitis admissions. Other causes include hyperlipidemia, drugs, toxins, infection, structural abnormalities, surgery, vascular
disease, trauma, hyperparathyroidism and hypercalcemia, renal transplantation, and hereditary pancreatitis. The most common cause of pancreatitis is alcohol abuse.

216
Q

Your patient with pancreatitis has a Ranson rule score of 8. The clinician should recognize that this is a risk of:
A. Pleural involvement
B. Alcoholism
C. High mortality
D. Bile duct obstruction

A

ANS: C
The Ranson rule uses a score determined by MRI results, with an index possible range of 0 to 10. A categorization of patients indicates the risk of both mortality and complication from pancreatitis.
Patients at the low end of the index (1–3) are predicted to have a low risk of mortality (3%) and complications (8%), whereas patients scoring at the high end (7–10) of the index are predicted to
have a higher incidence of mortality (17%) and/or complications (92%)

217
Q

Your patient complains of left upper quadrant pain, fever, extreme fatigue, and spontaneous bruising. The clinician should recognize that these symptoms are often related to:
A. Hematopoetic disorders
B. Hepatomegaly
C. Esophageal varices
D. Pleural effusion

A

ANS: A
LUQ pain can be associated with stomach or spleen disorders; however, it is often associated with causes that are outside the abdomen. Hematopoietic malignancies, such as lymphomas and leukemias, and other hematologic disorders, such as thrombocytopenia, polycythemia, myelofibrosis, and hemolyticanemia, often cause enlargement of the spleen, leading to LUQ pain. In addition to questions about the specific characteristics of the pain, it is important to ask the patient about fever, unusual bleeding or bruising, recent diagnosis of mononucleosis, fatigue, malaise, lymphadenopathy, cough, arthralgias, anorexia, weight loss, jaundice, high blood
pressure, and headache

218
Q

A 16-year-old patient presents with sore throat, cervical lymphadenopathy, fever, extreme fatigue,and left upper quadrant pain. The physical examination reveals splenomegaly. The clinician should recognize the probability of:
A. Bacterial endocarditis
B. Infectious mononucleosis
C. Pneumonia with pleural effusion
D. Pancreatic cancer

A

B

Hypersplenism is secondary to other primary disorders, most commonly cytopenic hematologic disorders, such as lymphoma, leukemia, thrombocytopenia, polycythemia, myelofibrosis, and
haemolytic anemias. With the sore throat and cervical lymphadenopathy, infection due to EpsteinBarr virus is common in adolescents. Infectious mononucleosis is an important disorder to
consider. Splenomegaly often occurs in infectious mononucleosis.

219
Q

Your patient complains of lower abdominal pain, anorexia, extreme fatigue, unintentional weight loss of 10 pounds in last 3 weeks, and you find a positive hemoccult on digital rectal examination.
Laboratory tests show iron deficiency anemia. The clinician needs to consider:
A. Diverticulitis
B. Appendicitis
C. Colon cancer
D. Peptic ulcer disease

A

ANS: C
A positive hemoccult on rectal examination may indicate an upper GI bleed or malignancy. Malignancy should also be suspected if there is weight loss and/or a palpable abdominal mass.

220
Q

Which of the following is the most common cause of heartburn-type epigastric pain?
A. Decreased lower esophageal sphincter tone
B. Helicobacteria pylori infection of stomach
C. Esophageal spasm
D. Excess use of NSAIDs

A

ANS: A
GERD is the most common organic cause of heartburn. GERD is caused by decreased lower esophageal sphincter (LES) tone. LES control can be decreased by several medications (e.g.,
theophylline, dopamine, diazepam, calcium-channel blockers), foods and/or beverages (caffeine, alcohol, chocolate, fatty foods), and tobacco use. When LES tone is lower than normal, secretions
are allowed to reflux into the esophagus, causing discomfort.

221
Q

A 22-year-old female enters the emergency room with complaints of right lower quadrant abdominal pain, which has been worsening over the last 24 hours. On examination of the abdomen, there is a palpable mass and rebound tenderness over the right lower quadrant. The clinician should recognize the importance of:
A. Digital rectal examination
B. Endoscopy
C. Ultrasound
D. Pelvic examination

A

ANS: D
A female with abdominal pain can have a GI or GU disorder or gynecologic problem. It is imperative to ask about the last menstrual period (LMP) and about birth control methods in order
to rule out ectopic pregnancy. A history of miscarriages and/or sexually transmitted diseases (STDs) can give more clues for the risk of ectopic pregnancy. Safe sex practices and the number of
sexual partners can alert the practitioner to the risk for pelvic inflammatory disease. No complaint of lower abdominal pain in a female should be evaluated without performing a pelvic examination.

222
Q

The major sign of ectopic pregnancy is:
A. Sudden onset of severe epigastric pain
B. Amenorrhea with unilateral lower quadrant pain
C. Lower back and rectal pain
D. Palpable abdominal mass

A

ANS: B
The most obvious sign of ectopic pregnancy is amenorrhea followed by spotting and sudden onset of severe lower quadrant pain. A stat pregnancy test should be performed. There is tenderness on
pelvic examination, and a pelvic mass may be palpated. Blood is present in the cul-de-sac. Shock and hemorrhage occur if the pregnancy ruptures. Abdominal distention with peritoneal signs will
ensue. Immediate laparoscopy or laparotomy is indicated because this condition is life threatening

223
Q

When ruptured ectopic pregnancy is suspected, the following procedure is most important:
A. Culdocentesis
B. CT scan
C. Abdominal x-ray
D. Digital rectal examination

A

A

The most obvious sign of ectopic pregnancy is amenorrhea followed by spotting and sudden onset of severe lower quadrant pain. A stat pregnancy test should be performed. The diagnosis of ectopic
pregnancy can be made with urine human chorionic gonadotropin (hCG) or stat serum hCG, pelvic ultrasound, and, if necessary, culdocentesis to detect blood in the cul-de-sac. There is tenderness on pelvic examination, and a pelvic mass may be palpated. Blood is present in the cul-de-sac. Shock and hemorrhage occur if the pregnancy ruptures. Abdominal distension with peritoneal signs will
ensue. Immediate laparoscopy or laparotomy is indicated because this condition is life threatening

224
Q

The majority of colon cancers are located in the:
A. Transverse colon
B. Cecum
C. Rectosigmoid region
D. Ascending colon

A

ANS: C
Colorectal cancer is the second leading cause of death from malignancies in the United States. Over half are located in the rectosigmoid region and are typically adenocarcinomas. Risk factors
include a history of polyps, positive family history of colon cancer or familial polyposis, ulcerative colitis, granulomatous colitis, and a diet low in fiber and high in animal protein, fat, and refined carbohydrates.

225
Q

The following symptom(s) in the patient’s history should raise the clinician’s suspicion of colon cancer:
A. Alternating constipation and diarrhea
B. Narrowed caliber of stool
C. Hematochezia
D. All of the above

A

ANS: D
Colon cancer may be present for several years before symptoms appear. Complaints include fatigue, weakness, weight loss, alternating constipation and diarrhea, a change in the caliber of
stool, tenesmus, urgency, and hematochezia. Physical examination is usually normal except in advanced disease, when the tumor can be palpated or hepatomegaly is present, owing to metastatic
disease

226
Q

A patient presents to the emergency department with nausea and severe, colicky back pain that radiates into the groin. When asked to locate the pain, he points to the right costovertebral angle
region. His physical examination is unremarkable. Which of the following lab tests is most important for the diagnosis?
A. Urinalysis
B. Serum electrolyte levels
C. Digital rectal exam
D. Lumbar x-ray

A

ANS: A
Urinary calculi can occur anywhere in the urinary tract; therefore, pain can originate in the flank or kidney area and radiate into the RLQ or LLQ and then to the suprapubic area as the stone attempts
to move down the tract. The pain is severe, acute, and colicky and may be accompanied by nausea and vomiting. If the stone becomes lodged at the ureterovesical junction, the patient will complain
of urgency and frequency. Blood will be present in the urine.

227
Q

Your 34-year-old female patient complains of a feeling of “heaviness” in the right lower quadrant, achiness, and bloating. On pelvic examination, there is a palpable mass in the right lower quadrant.
Urine and serum pregnancy tests are negative. The diagnostic tool that would be most helpful is:
A. Digital rectal exam
B. Transvaginal ultrasound
C. Pap smear
D. Urinalysis

A

ANS: B
Ovarian masses are often asymptomatic, but symptoms may include pressure-type pain, heaviness, aching, and bloating. Masses are typically detected on pelvic examination. In advanced
malignancies, ascites is often present. An elevated cancer antigen 125 (CA-125) result indicates the likelihood that the mass is malignant. A transvaginal pelvic ultrasound has a higher diagnostic
sensitivity than transabdominal ultrasound. If diagnosis is unclear, CT, MRI, or PET scan can be performed. A laparoscopy or exploratory laparotomy is necessary for staging, tumor debulking, and resection

228
Q

Your 54-year-old male patient complains of a painless “lump” in his lower left abdomen that comes and goes for the past couple of weeks. When examining the abdomen, you should have the
patient:
A. Lie flat and take a deep breath
B. Stand and bear down against your hand
C. Prepare for a digital rectal examination
D. Lie in a left lateral recumbent position

A

ANS: B
In the majority of hernia cases, a history of heavy physical labor or heavy lifting can be elicited. Right or left lower quadrant pain that may radiate into the groin or testicle is typical. The pain is
usually dull or aching unless strangulated, in which case the pain is more severe. The pain increases with straining, lifting, or movement of the lower extremities. Physical examination includes palpating the femoral area and inguinal ring for bulging or tenderness. Ask the patient to bear down against your hand

229
Q

A nurse practitioner reports that your patient’s abdominal x-ray demonstrates multiple air-fluid levels in the bowel. This is a diagnostic finding found in:
A. Appendicitis
B. Cholecystitis
C. Bowel obstruction
D. Diverticulitis

A

ANS: C
The most common causes of mechanical obstruction are adhesions, almost exclusively in patients with previous abdominal surgery, hernias, tumors, volvulus, inflammatory bowel disease (Crohn’s
disease, colitis), Hirschsprung’s disease, fecal impaction, and radiation enteritis. Initially, the patient complains of a cramping periumbilical pain that eventually becomes constant. Physical
examination reveals mild, diffuse tenderness without peritoneal signs, and possibly visible peristaltic waves. In early obstruction, tinkles, rushes, and borborygmi can be heard. In late
obstruction, bowel sounds may be absent. The diagnosis can be made with flat and upright abdominal films looking for bowel distension and the presence of multiple air-fluid levels. CT or
MRI may be necessary for confirmation.

230
Q

A 76-year-old patient presents to the emergency department with severe left lower quadrant abdominal pain, diarrhea, and fever. On physical examination, you note the patient has a positive
heel strike, and left lower abdominal rebound tenderness. These are typical signs and symptoms of which of the following conditions?
A. Diverticulitis
B. Salpingitis
C. Inflammatory bowel disease
D. Irritable bowel syndrome

A

ANS: A
Diverticular disease is prevalent in patients over 60 years of age. Since the sigmoid colon has the smallest diameter of any portion of the colon, it is the most common site for the development of diverticula. Although the pain can be generalized, it is typically localized to the left lower abdomen and is accompanied by tenderness, fever, and leukocytosis. Other symptoms can include constipation or loose stools, nausea, vomiting, and positive stool occult blood. With diverticulitis, there is an increased risk of perforation, which presents with a more dramatic clinical picture as a result of peritonitis. Look for signs of peritonitis, such as a positive heel strike test and/or rebound tenderness

231
Q

Which of the following conditions is the most common cause of nausea, vomiting, and diarrhea?
A. Viral gastroenteritis
B. Staphylococcal food poisoning
C. Acute hepatitis A
D. E.coli gastroenteritis

A

ANS: A
Viral gastroenteritis is the most common cause of nausea, vomiting, and diarrhea. At least 50% of cases of gastroenteritis as foodborne illness are due to norovirus. Another 20% of cases, and the
majority of severe cases in children, are due to rotavirus. Other significant viral agents include adenovirus and astrovirus.

232
Q

A patient presents to the emergency department with complaints of vomiting and abdominal pain. You note that the emesis contains bile. On physical examination, there is diffuse tenderness,
abdominal distension, and rushing, high-pitched bowel sounds. Which of the following diagnoses would be most likely?
A. Gastric outlet obstruction
B. Small bowel obstruction
C. Distal intestinal blockage
D. Colonic obstruction

A

b

The contents of the vomitus commonly vary according to the level of obstruction. Gastric outlet obstruction is associated with emesis containing undigested food. Proximal small intestinal blockage is likely to be bile-stained. Distal intestinal blockage is more likely to contain fecal matter. The degree of cramping and pain is often related to the proximity of the obstruction, so that obstructions of the lower intestines may have less severe cramping, vomiting, and/or pain. Bowel sounds often are high pitched and metallic sounding but may later become absent. Tenderness may be localized or diffuse. Distention as well as a succussion splash may be present.

233
Q

Your 5-year-old female patient presents to the emergency department with sore throat, vomiting, ear ache, 103 degree fever, photophobia, and nuchal rigidity. She has an episode of projectile
vomiting while you are examining her. The clinician should recognize that the following should be done:
A. Abdominal x-ray
B. Fundoscopic examination
C. Lumbar puncture
D. Analysis of vomitus

A

ANS: C
The range of neurologic disorders that result in nausea and/or vomiting is broad. Included are meningitis, increased intracranial pressure (ICP), migraines, a space-occupying lesion, and
Ménière’s disorder. Central nervous system-related vomiting is often projectile and may not be preceded by nausea. Papilledema may accompany increased ICP. Neurological deficits may be evident with increased ICP, space-occupying lesions, and meningitis. Nuchal rigidity is a classic finding for meningitis.

234
Q

A 9-year-old boy accompanied by his mother reports that since he came home from summer camp, he has had fever, nausea, vomiting, severe abdominal cramps and watery stools that contain blood
and mucus. The clinician should recognize the importance of:
A. Stool for ova and parasites
B. Abdominal x-ray
C. Stool for clostridium
D. Fecal occult blood test

A

ANS: A
Parasites causing diarrhea usually enter the body through the mouth. They are swallowed and can remain in the intestine or burrow through the intestinal wall and invade other organs. Certain parasites, most commonly Giardia lamblia, transmitted by fecally contaminated water or food, can cause diarrhea, bloating, flatulence, cramps, nausea, anorexia, weight loss, greasy stools because of
its interference with fat absorption, and occasionally fever. Symptoms usually occur about 2 weeks after exposure and can last 2 to 3 months. Often, the symptoms are vague and intermittent, which
makes diagnosis more difficult. Serial stool samples for O&P should be ordered because a single sample may not reveal the offending parasite.

235
Q

A 56-year-old male complains of anorexia, changes in bowel habits, extreme fatigue, and unintentional weight loss. At times he is constipated and other times he has episodes of diarrhea.
His physical examination is unremarkable. It is important for the clinician to recognize the importance of:
A. CBC with differential
B. Stool culture and sensitivity
C. Abdominal x-ray
D. Colonoscopy

A

ANS: D
The symptoms and severity of the diarrhea vary according to the underlying cause. The symptoms of carcinomas are generally insidious. The diarrhea is mild and intermittent. Often malignancies
are found on routine hemoccults, sigmoidoscopy, or colonoscopy. There should be a high index of suspicion with unexplained weight loss or new-onset iron-deficiency anemia in a patient over 40
years old.

236
Q

A 20-year-old engineering student complains of episodes of abdominal discomfort, bloating, and episodes of diarrhea. The symptoms usually occur after eating, and pain is frequently relieved with bowel movement. She is on a “celiac diet” and the episodic symptoms persist. Physical examination and diagnostic tests are negative. Colonoscopy is negative for any abnormalities. This
is a history and physical consistent with:
A. Inflammatory bowel disease
B. Irritable bowel syndrome
C. Laxative abuse
D. Norovirus gastroenteritis

A

B

Irritable bowel syndrome (IBS) is a functional bowel disorder characterized by mild to severe abdominal pain, discomfort, bloating, and alteration of bowel habits. The exact cause is unknown.
In some cases, the symptoms are relieved by bowel movements. Diarrhea or constipation may predominate, or they may be mixed (classified as IBS-D, IBS-C, or IBS-M, respectively). IBS may
begin after an infection (postinfectious, IBS-PI) or a stressful life event. IBS is a motility disorder involving the upper and lower GI tracts that causes intermittent nausea, abdominal pain and
distention, flatulence, pain relieved by defecation, diarrhea, and/or constipation. Symptoms usually occur in the waking hours and may be worsened or triggered by meals. It is three times more
prevalent in women, accounts for more than half of all GI referrals, and is highly correlated with emotional factors, particularly anxiety and stress.

237
Q

. A 78-year-old female patient is suffering from heart failure, GERD, diabetes, and depression. She presents with complaints of frequent episodes of constipation. Her last bowel movement was 1
week ago. Upon examination, you palpate a hard mass is the left lower quadrant of the abdomen. You review her list of medications. Which of the following of her medications cause constipation?
A. Digitalis (Lanoxin)
B. Amlodipine (Norvasc)
C. Sertraline (Zoloft)
D. Metformin (Glucophage)

A

. ANS: B
Medications that frequently cause constipation include:
- Analgesics/narcotics
- Antacids containing aluminum
- Anticonvulsants
- Antidepressants
- Antihypertensives (calcium-channel blockers, beta blockers)
- Antiparkinsonism agents
- Antispasmodics
- Calcium supplements
- Diuretics
- Iron supplements
- Sedatives/tranquilizers

238
Q

You are examining a 55-year-old female patient with a history of alcohol abuse. She complains of anorexia, nausea, pruritus, and weight loss over the last month. On physical examination, you note
yellow hue of the skin and sclera. Which of the following physical examination techniques is most important?
A. Scratch test
B. Heel strike
C. Digital rectal examination
D. Pelvic examination

A

ANS: A
Cirrhosis develops with the replacement of normal liver tissue by regenerative, fibrotic nodules and may occur in the late phase of a variety of disorders that damage the liver, such as alcohol
toxicity. A patient may present with jaundice and describe an associated, progressive pattern of pruritus, weakness, anorexia, nausea, and weight loss. Determine the size and consistency of the
liver as well as any tenderness. The scratch test is a method used to ascertain the location and size of a patient’s liver during a physical assessment. The scratch test uses auscultation to detect the
differences in sound transmission through the abdominal cavity over solid and hollow organs and spaces. After placing a stethoscope over the approximate location of a patient’s liver, the examiner will then scratch the skin of the patient’s abdomen lightly, moving laterally along the liver border. When the liver is encountered, the scratching sound heard in the stethoscope will increase significantly. In this manner, the size and shape of a patient’s liver can be ascertained

239
Q

You observe Charcot’s triad of sign and symptoms in a patient under your care. This is commonly seen in which of the following disorders?
A. Cirrhosis
B. Pancreatitis
C. Cholangitis
D. Portal hypertension

A

c

Occlusion of the common bile duct may occur with disorders of the gallbladder and/or bile duct, such as cholecystitis, cholelithiasis, and cholangitis. All three conditions are generally accompanied by RUQ discomfort, anorexia, and nausea. Charcot’s triad, which includes jaundice, RUQ pain, and fever/chills, is common to problems resulting in obstructions of the bile duct.

240
Q

A 59-year-old patient with history of alcohol abuse is admitted for hematemesis. On physical examination, you note ascites and caput medusa. A likely cause for the hematemesis is:
A. Peptic ulcer disease
B. Barrett’s esophagus
C. Pancreatitis
D. Esophageal varices

A

ANS: D
Patients with portal hypertension may develop GI bleeding from varices of the esophagus, stomach, intestines, or other sites. Portal hypertension is most commonly associated with cirrhosis, usually caused by alcohol abuse or hepatitis. Check for signs of liver disease, including jaundice, cirrhosis, telangiectasia, hepatomegaly, and RUQ tenderness. Ascites occurs due to venous congestion. Caput medusa is the distension of paraumbilical veins due to portal hypertension.

241
Q

A 16-year-old female with anorexia and bulimia is admitted for hematemesis. She admits to inducing vomiting often. On physical examination, you note pallor, BMI less than 15, and
hypotension. A likely reason for hematemesis is:
A. Mallory-Weisstear
B. Cirrhosis
C. Peptic ulcer disease
D. Esophageal varies

A

. ANS: A
Upper GI hemorrhage may result from a tear at the gastroesophageal junction, known as a Mallory-Weiss tear. A patient may develop more than one tear. These tears are most common in
alcoholic or bulimic patients following repeated episodes of vomiting or severe retching. If a laceration/tear of the mucosa causes GI bleeding, the patient may demonstrate alterations in hemodynamic status

242
Q

An 82-year-old female presents to the emergency department with epigastric pain and weakness. She admits to having dark, tarry stools for the last few days. She reports a long history of pain due
to osteoarthritis. She self-medicates daily with ibuprofen, naprosyn, and aspirin for joint pain. On physical examination, she has orthostatic hypotension and pallor. Fecal occult blood test is
positive. A likely etiology of the patient’s problem is:
A. Mallory-Weiss tear
B. Esophageal varices
C. Gastric ulcer
D. Colon cancer

A

ANS: C
Bleeding occurs after an area of gastric mucosal injury has ulcerated. Explore symptoms of epigastric and/or periumbilical discomfort. Identify potential causes of gastric mucosal injury—the
most common being NSAID use and stress. Many elderly individuals self-medicate with over-thecounter aspirin preparations and various NSAIDs. Commonly, they use too many medications that
have side effects of gastric irritation.

243
Q

A 48-year-old male presents to the clinic with complaints of anorexia, nausea, weakness, and unintentional weight loss over the last few weeks. On physical examination, the patient has
jaundice of the skin as well as sclera and a palpable mass in the epigastric region. In addition to CBC and bilirubin levels, all of the following tests would be helpful except:
A. Liver enzymes
B. Amylase
C. Lipase
D. Uric acid

A

ANS: D
Primary or metastatic cancers of the liver and/or pancreas can cause obstructivehyperbilirubinemia and jaundice. Jaundice may be the initial sign of a malignancy or may follow the development of other symptoms. Ask about associated symptoms, such as RUQ discomfort, nausea, fever, back pain, weight loss, fatigue/weakness, and pruritus. None of these symptoms are specific to malignancy; however, other causes of jaundice are less likely to be associated with weight loss. During the abdominal examination, carefully palpate the area of the liver and the remainder of the abdomen, checking for masses or unexpected findings. In addition to a CBC, liver functions, amylase, lipase, and bilirubin levels, abdominal CT and/or ultrasound should be ordered promptly

244
Q

Your 66-year-old male patient complains of weakness, fatigue, chronic constipation for the last month, and dark stools. On CBC, his results show iron deficiency anemia. Colon cancer is diagnosed. Which of the following laboratory tests is used to follow progress of colon cancer?
A. Alpha fetoprotein (AFP)
B. Carcinogenic embryonic antigen (CEA)
C. Carcinoma antigen 125 (CA-125)
D. Beta-human chorionic gonadotropin (beta HCG)

A

B

AFP can help diagnose and guide the treatment of liver cancer (hepatocellular carcinoma). CA-125 is the standard tumor marker used to follow women during or after treatment for epithelial ovarian
cancer (the most common type of ovarian cancer) as well as fallopian tube cancer and primary peritoneal cancer. Serum beta HCG is a pregnancy marker. CEA is not used to diagnose or screen
for colorectal cancer, but it’s the preferred tumor marker to help predict outlook in patients with colorectal cancer. The higher the CEA level at the time colorectal cancer is detected, the more likely it is that the cancer is advanced.

245
Q

Your patient is a 33-year-old female gave birth last week. She complains of constipation, rectal pain, and itching. She reports bright red blood on the toilet tissue. The clinician should recognize
the need for:
A. Digital rectal exam
B. CEA blood test
C. Colonoscopy
D. Fecal occult blood test

A

ANS: A
The most common cause of lower GI bleeding is hemorrhoids. The bleeding associated with hemorrhoids is usually evident as red blood on the formed stool, in the toilet bowl, or on the toilet tissue following a bowel movement. Patients with hemorrhoids often complain of rectal discomfort as well as the contributing factors for hemorrhoid development, including constipation. Inspect the
perianal rectal tissue. Anoscopy may be indicated. Perform a digital rectal examination to assess internal haemorrhoids.

246
Q

Part of the diagnostic testing to rule in/out a DVT.

A

D-dimer

247
Q

An enzyme released when organ or tissue is destroyed, particularly myocardial tissue. Can also be elevated in hemolytic states, hyperthyroidism, kidney disease, gastric malignancy, and megaloblastic anemia.

A

Lactate dehydrogenase (LDH)

248
Q

This term describes sinus arrest, sinoatrial block, and persistent sinus bradycardia of unknown origin. Often caused or exacerbated by drugs, particularly digitalis, calcium channel blockers, beta blockers, and antiarrhythmics.

A

Sick sinus syndrome

249
Q

Diagnostic Studies for arrhythmias

A

EKG

Holter monitor

Electrolytes

Thyroid functions

Cardiac enzyme studies

Medication levels

250
Q

can assist in diagnosing CAD and angina and establishing safe exercise levels. The addition of the radioactive substance thallium helps determine the extent of coronary artery blockage and blood flow both at rest and with exercise.

A

The stress test

251
Q

chest pain diagnostic studies

A

stress test
echo
cardia enzymes
cardiac cath
EKG
cbc
cxr

251
Q

grades of murmurs

Grade I—
Grade II—
Grade III—
Grade IV—
Grade V—
Grade VI—

A

Grade I—very faint, not heard in all positions
Grade II—soft but easily heard
Grade III—moderately loud
Grade IV—loud, may be associated with a thrill
Grade V—very loud, may be heard with the stethoscope barely on the chest, associated with a thrill
Grade VI—may be heard with the stethoscope off the chest, associated with a thrill

252
Q

are systolic murmurs caused by a temporary increase in blood flow rather than by a structural abnormality and include such conditions as anemia, hyperthyroidism, pregnancy, and fever.

A

Physiological or functional murmurs

252
Q

Squatting
Worsens murmur of ____________
Improves murmur of ___________

A

Squatting
Worsens murmur of aortic stenosis
Improves murmur of mitral valve stenosis

252
Q

Grade I—very faint, not heard in all positions
Grade II—soft but easily heard
Grade III—moderately loud
Grade IV—loud, may be associated with a thrill
Grade V—very loud, may be heard with the stethoscope barely on the chest, associated with a thrill
Grade VI—may be heard with the stethoscope off the chest, associated with a thrill

A
253
Q

Valsalva
Decreases murmur of ________

A

aortic stenosis

254
Q

_______ murmurs almost always indicate pathology. They are low-pitched and often very faint and difficult to hear.

A

diastolic

255
Q

pulse-

Arterial insufficiency:
Venous insufficiency:

A

Arterial: absent or decresed
Venous: normal

256
Q

edema
Arterial *insufficiency**:
Venous insufficiency:

A

Arterial: absent or mild
Venous: significant

257
Q

pain
Arterial insufficiency:
Venous insufficiency:

A

Arterial: severe
Venous: absent or mild

258
Q

temp

Arterial insufficiency:
Venous insufficiency:

A

Arterial: cool
Venous: normal

259
Q

color

Arterial insufficiency:
Venous insufficiency:

A

Arterial: Pallor with elevation; dusky red on dependency

Venous: Hyperpigmented; cyanotic on dependency

260
Q

skin

Arteria insufficiencyl:
Venous insufficiency:

A

Arterial:Thin, atrophic; risk of gangrene

Venous:
Thick; risk of stasis ulcers

261
Q

dvt risk factors

A

immobility, orthopedic surgery, malignancy, HF, smoking, pregnancy, oral contraceptive or hormone use, advanced age, and clotting disorders

262
Q

heart failure that

Subjective Data
* Peripheral edema, particularly at the end of the day or after prolonged sitting

  • Weight gain

Objective Data
* Pitting edema in lower extremities

  • Jugular venous distention
  • Ascites
  • Hepatomegaly
A

right sided

263
Q

*

heart failure that

  • Fatigue
  • Breathing difficulty, shortness of breath
  • Orthopnea
  • Exercise intolerance

Objective Data
* Sudden with acute pulmonary edema or gradual symptom onset

  • Crackles on pulmonary examination
A

left sided

264
Q

Subjective Data
* Sharp and stabbing chest pain

  • Pain worse with coughing, swallowing, deep breathing or lying flat, or movement
  • Pain may be most severe when supine, relieved by sitting up and leaning forward
  • Pain in the back, neck, or left shoulder
  • Difficulty breathing when lying down
  • Dry cough
  • Anxiety or fatigue

Objective Data
* Scratchy, grating, triphasic friction rub on auscultation, comprises ventricular systole, early diastolic ventricular filling, and late diastolic atrial systole

  • Friction rub easily heard just left of the sternum in third and fourth intercostal spaces
A

Pericarditis

265
Q

subjective Data
* Anxiety, restlessness

  • Chest pain
  • Difficulty breathing
  • Discomfort, sometimes relieved by sitting upright or leaning forward
  • Syncope, lightheadedness
  • Pale, gray, or blue skin
  • Palpitations
  • Rapid breathing
  • Swelling of the abdomen or arms or neck veins

Objective Data
* Beck triad (jugular venous distention, hypotension, and muffled heart sounds)

A

Cardiac Tamponade

Excessive accumulation of effused fluids or blood between the pericardium and the heart

266
Q

Subjective Data
* Fatigue
* Tachypnea
* Exertional dyspnea
* Cough
* Hemoptysis
* Lightheadedness
* Syncope

Objective Data
* Evidence of pulmonary disease
* Wheezes and crackles on auscultation
* Increase in chest diameter
* Labored respiratory efforts with chest wall retractions
* Evidence of right heart failure and hypertrophy
* Distended neck veins with prominent A or V waves
* Cyanosis
* Left parasternal systolic heave
* Loud S2 exaggerated in the pulmonic region
* Lower extremity edema

A

Cor Pulmonale
Enlargement of the right ventricle secondary to chronic lung disease

267
Q

Subjective Data
* Deep substernal or visceral pain that often radiates to the jaw, neck, and left arm

  • Discomfort may be mild, especially in older adults or patients with diabetes mellitus
  • Nausea
  • Fatigue
  • Shortness of breath

Objective Data
* Dysrhythmias are common.

  • S4 is usually present.
  • Distant heart sounds
  • Soft, systolic, blowing apical murmur
  • Thready pulse
  • Blood pressure varies, although hypertension is usual in the early phases.
  • New ST elevation in two contiguous leads
A

Myocardial Infarction
Ischemic myocardial necrosis caused by abrupt decrease in coronary blood flow to a segment of the myocardium

268
Q

murmur timing and duration

Begins with S1, decrescendos, ends well before S2

A

Early systolic

269
Q

murmur timing and duration

Begins after S1, ends before S2; crescendo-decrescendo quality sometimes difficult to discern

A

Midsystolic (ejection)

270
Q

murmur timing and duration

Begins mid to late systole, crescendos, ends at S2; often introduced by mid to late systolic clicks

A

Late systolic

271
Q

murmur timing and duration

Begins with S2

A

Early diastolic

272
Q

murmur timing and duration

Begins at clear interval after S2
A

Middiastolic

273
Q

murmur timing and duration

Begins immediately before S1

A

Late diastolic (presystolic)

274
Q

murmur timing and duration

Begins with S1, occupies all of systole, ends at S2

A

Holosystolic (pansystolic)

275
Q

murmur timing and duration

Begins with S2, occupies all of diastole, ends at S1

A

Holodiastolic (pandiastolic)

276
Q

murmur timing and duration

Starts in systole, continues without interruption through S2, into all or part of diastole; does not necessarily persist throughout entire cardiac cycle

A

Continuous

277
Q

Heard with bell at apex, patient in left lateral decubitus position

A

Mitral Stenosis

278
Q

Low-frequency diastolic rumble, more intense in early and late diastole, does not radiate; systole usually quiet; palpable thrill at apex in late diastole common; S1 increased and often palpable at left sternal border; S2 split often with accented P2; opening snap follows P2 closely

Visible lift in right parasternal area if right ventricle hypertrophied
Arterial pulse amplitude decreased

A

Mitral Stenosis

279
Q

Heard over aortic area; ejection sound at second right intercostal border

A

Aortic Stenosis

280
Q

Midsystolic (ejection) murmur, medium pitch, coarse, diamond-shaped, a crescendo-decrescendo; radiates along left sternal border (sometimes to apex) and to carotid with palpable thrill; S1 often heard best at apex, disappearing when stenosis is severe, often followed by ejection click; S2 soft or absent and may not be split; S4 palpable; ejection sound muted in calcified valves; the more severe the stenosis, the later the peak of the murmur in systole

Apical thrust shifts down and left and is prolonged if left ventricular hypertrophy is also present.

A

Aortic Stenosis

281
Q

Heard at apex and along left sternal border

A

Subaortic Stenosis

282
Q

Murmur fills systole, diamond-shaped, medium pitch, coarse; thrill often palpable during systole at apex and right sternal border; multiple waves in apical impulses; S2 usually split; S3 and S4 often present

Arterial pulse brisk, double wave in carotid common; jugular venous pulse prominent

A

Subaortic Stenosis

283
Q

Heard over pulmonic area radiating to left and into neck; thrill in second and third left intercostal spaces

A

Pulmonic Stenosis

284
Q

Systolic (ejection) murmur
medium pitch, coarse; usually with thrill; S1 often followed quickly by ejection click; S2 often diminished
usually wide split
common in right ventricular hypertrophy

A

Pulmonic Stenosis

285
Q

Heard with bell over tricuspid area

A

Tricuspid Stenosis

286
Q

Diastolic rumble accentuated early and late in diastole, resembling mitral stenosis but louder on inspiration; diastolic thrill palpable over right ventricle; S2 may be split during inspiration

Arterial pulse amplitude decreased; jugular venous pulse prominent, especially a wave; slow fall of V wave

A

Tricuspid Stenosis

287
Q
A
288
Q

Heard best at apex; loudest there, transmitted into left axilla

A

Mitral Regurgitation

289
Q

Holosystolic, plateau-shaped intensity, high pitch, harsh blowing quality, often quite loud and may obliterate S2; radiates from apex to base or to left axilla; thrill may be palpable at apex during systole; S1 intensity diminished; S2 more intense with P2 often accented; S3 often present; S3–S4 gallop common in late disease

If mild, late systolic murmur crescendos; if severe, early systolic intensity decrescendos; apical thrust more to left and down in ventricular hypertrophy

A

Mitral Regurgitation

290
Q

Heard at apex and left lower sternal border; easily missed in supine position; also listen with patient upright

A

Mitral Valve Prolapse

291
Q

Typically late systolic murmur preceded by midsystolic clicks, but both murmur and clicks highly variable in intensity and timing

A

Mitral Valve Prolapse

292
Q

Heard with diaphragm, patient sitting and leaning forward; Austin-Flint murmur heard with bell; ejection click heard in second intercostal space

A

Aortic Regurgitation

293
Q

Early diastolic, high pitch, blowing, often with diamond-shaped midsystolic murmur, sounds often not prominent; duration varies with blood pressure; low-pitched, rumbling murmur at apex common (Austin-Flint); early ejection click sometimes present; S1 soft; S2 split may have drumlike quality; Mitral (M1) and A2 often intensified, S3–S4 gallop common

In left ventricular hypertrophy, prominent prolonged apical impulse down and to left

Pulse pressure wide; water-hammer or Corrigan pulse common in carotid, brachial, and femoral arteries

A

Aortic Regurgitation

294
Q

Difficult to distinguish from aortic regurgitation on physical examination

A

Pulmonic Regurgitation

295
Q

Heard at left lower sternum, occasionally radiating a few centimeters to left

A

Tricuspid Regurgitation

296
Q

Holosystolic murmur over right ventricle blowing
increased on inspiration
S3 and thrill over tricuspid area
common In pulmonary hypertension visible lift to right of sternum
Jugular venous pulse has large V waves

A

Tricuspid Regurgitation

297
Q

Subjective Data
* Usually affects persons older than 50 years of age

  • Flu-like symptoms (e.g., low-grade fever, malaise, anorexia) may be accompanied by polymyalgia rheumatica involving the hips, neck, and shoulders
  • Headache in the temporal region on one or both sides, although the headache can occur in other regions
  • Ocular symptoms, including loss of vision, are common
  • Ischemia can also cause tongue pain and jaw claudication

Objective Data
* Area over the temporal artery may be red, swollen, tender, and nodular

  • Temporal pulse may be strong, weak, or absent
A

Temporal Arteritis (Giant Cell Arteritis)

An inflammatory disease of the branches of the aortic arch, including the temporal arteries

298
Q

Subjective Data
* Generally asymptomatic until they rupture, dissect, or compress an adjacent structure

  • With dissection, the patient may describe a severe ripping pain.

Objective Data
* Pulsatile swelling along the course of an artery

  • A thrill or bruit may be evident over the aneurysm.
A

Arterial Aneurysm

A localized dilation, which is generally defined as 1.5 times the diameter of the normal artery, caused by a weakness in the arterial wall

299
Q

subjective Data
* Patients may present with lower extremity edema, varicose veins, or claudication due to ischemia.

Objective Data
* A continuous bruit or thrill over the area of the fistula suggests its presence.

  • Edema or ischemia may develop in the involved extremity.
A

Arteriovenous Fistula
A pathologic communication between an artery and a vein

300
Q

Subjective Data
* Pain in muscle after exercise that disappears with rest

  • Amount of exercise needed to cause discomfort is predictable (e.g., occurring each time the same distance is walked)

Objective Data
* In claudication, the limb appears healthy, but pulses are weak or absent.

  • Progressive stenosis results in severe ischemia, in which the foot or leg is painful at rest, is cold and numb, and has skin changes (e.g., dry and scaling, with poor hair and nail growth).
  • Edema seldom accompanies this disorder, but ulceration is common in severe disease, and the muscles may atrophy.
A

Peripheral Arterial Disease

Stenosis of the blood supply to the extremities by atherosclerotic plaques

301
Q

Liver and gallbladder
Pylorus
Duodenum
Head of pancreas
Right adrenal gland
Portion of right kidney
Hepatic flexure of colon
Portions of ascending and transverse colon

A

Right Upper Quadrant (RUQ)

302
Q

Spleen
Stomach
Body of pancreas
Left adrenal gland
Portion of left kidney
Splenic flexure of colon
Portions of transverse and descending colon

A

Left Upper Quadrant (LUQ)

303
Q

Initially periumbilical or epigastric; colicky; later becomes localized to RLQ

Guarding, tenderness; + iliopsoas and + obturator signs, RLQ skin hyperesthesia; anorexia, nausea, or vomiting after onset of pain; low-grade fever; + Aaron, Rovsing, Markle, and McBurney signs a

A

Appendicitis

304
Q

Onset sudden or gradual; pain generalized or localized, dull or severe, and unrelenting; guarding; pain on deep inspiration

Shallow respiration; + Blumberg, Markle, and Ballance signs; reduced or absent bowel sounds, nausea and vomiting; + obturator + iliopsoas signs
A

Peritonitis

305
Q

Severe, unrelenting RUQ or epigastric pain; may be referred to right subscapular area

RUQ tenderness and rigidity; + Murphy sign, palpable gallbladder, anorexia, vomiting, fever, possible jaundice

A

Cholecystitis

306
Q

Dramatic, sudden, excruciating LUQ, epigastric, or umbilical pain; may be present in one or both flanks; may be referred to left shoulder and penetrates to back

Epigastric tenderness, vomiting, fever, shock; + Grey Turner sign; + Cullen sign: both signs occur 2–3 days after onset

A

Pancreatitis

307
Q

Epigastric, radiating down left side of abdomen especially after eating; may be referred to back

Flatulence, borborygmus, diarrhea, dysuria, tenderness on palpation

A

Diverticulitis

308
Q

Abrupt, severe, colicky, spasmodic; referred to epigastrium, umbilicus

Distention, minimal rebound tenderness, vomiting, localized tenderness, visible peristalsis; bowel sounds absent (with paralytic obstruction) or hyperactive high pitched (with mechanical obstruction)

A

Intestinal obstruction

309
Q

Ask the patient to stand with straight legs and to raise up on toes. Then ask the patient to relax, allowing the heel to strike the floor, thus jarring the body. A positive heel strike is indicative of appendicitis and peritoneal irritation. Alternatively, strike the plantar surface of the heel with your fist while the patient rests supine on the examination table.

A

Heel Strike

310
Q

Pain is elicited in appendicitis by inward rotation of the hip with the knee bent so that the obturator internus muscle is stretched.

A

Obturator Sign

311
Q

Place your hand on the patient’s thigh just above the knee and ask the patient to raise the thigh against your hand. This contracts the psoas muscle and produces pain in patients with an inflamed appendix

A

Psoas Sign

312
Q

Pain is present on deep inspiration when pressing the fingers under the rib cage. positive in cholecystitis.

A

Murphy’s Sign

313
Q
A